Расчет пути, скорости и времени движения: равномерное и неравномерное

 

Равномерное движение, это вдвижение спостоянной скоростью. То есть другимим словами, тело за одинаковые промежутки времени должно проходить одинаковое расстояние. Например, если машина будет за каждый час своего пути проезжать расстояние в 50 километров, то такое движение будет являться равномерным.

Обычно равномерное движение очень редко можно встретить в реальной жизни. За примеры равномерного движения в природе, можно считать вращение Земли вокруг Солнца. Или например, конец секундной стрелки часов, тоже будет двигаться равномерно.

Расчет скорости при равномерном движении

Скорость тела при равномерном движении будет вычисляться по следующей формуле.

  • Скорость = путь / время.

Если обозначить скорость движения буквой V, время движения буквой t, а путь пройденный телом буквой S, то получим следующую формулу.

Единица измерения скорости 1 м/с. То есть тело проходит расстояние в один метр, за время равное одной секунде.

Движения с переменной скоростью называется неравномерным движением. Чаще всего, все тела в природе двигаются именно неравномерно. Например, человек, когда куда-либо идет, двигается неравномерно, то есть его скорость в течении всего пути будет изменяться.

Расчет скорости при неравномерном движении

При неравномерном движении, скорость все время изменяется, и в этом случае говорят о средней скорости движения.

Средняя скорость неравномерного движения вычисляется по формуле

Из формулы для определения скорости, мы можем получить и другие формулы, например, для расчета пройденного пути или времени, которое двигалось тело.

Расчет пути при равномерном движении

Чтобы определить путь, который прошло тело при равномерном движении, необходимо скорость движения тела умножить на время которое это тело двигалось.

То есть, зная скорость и время движения, мы всегда сможем найти путь.

Теперь, получим формулу для расчета времени движения, при известных: скорости движения и пройденном пути.

Расчет времени при равномерном движении

Для того чтобы определить время равномерного движения, необходимо путь пройденный телом, поделить на скорость, с которой это тело двигалось.

Полученные выше формулы будут справедливы, если тело совершало равномерное движение. 

При расчете  средней скорости неравномерного движения, полагают, что движение было равномерным. Исходя из этого, для вычисления по средней скорости неравномерного движения, пути или времени движения используют те же самые формулы, что и при равномерном движении.

Расчет пути при неравномерном движении

Получаем, что путь пройденный телом при неравномерном движении, равен произведению средней скорости на время которое тело двигалось.

Расчет времени при неравномерном движении

Время необходимое для прохождения некоторого пути при неравномерном движении, равняется частному от деления пути на среднюю скорость неравномерного движения.{2}} d t(3)$$

Местоположение перемещающейся материальной точки в фиксированный момент времени, например t=t1 называют начальным положением. Очень часто полагают t1=0. Длин пути, который прошла материальная точка из начального положения – скалярная функция времени: s=s(t).

Считают, что за промежуток времени $d t \rightarrow 0$ материальная точка проходит путь ds, который называют элементарным. При этом:

$$d s=|d \bar{r}|=v d t$$

где $\bar{r}$ – вектор элементарного перемещения материальной точки, v – модуль скорости ее движения.

Виды движения и формулы длины пути

Длина пути при равномерном движении (v=const) точки равна:

$$s=v\left(t_{2}-t_{1}\right)(5)$$

где t1 – начало отсчета движения, t2 – окончание отсчета. Формула (5) показывает то, что длина пути, который проходит равномерно движущаяся материальная точка – это линейная функция времени.

Если движение не является равномерным, то можно длину пути $\Delta s$ на отрезке времени от $t$ до $t + \Delta t$ находят как:

$$\Delta s=\langle v\rangle \Delta t(6)$$

где $\langle v\rangle$ – средняя путевая скорость.{2}}{2}(7)$$

где a – постоянное ускорение, v0 – начальная скорость движения.

Единицы измерения пути

Основной единицей измерения пути в системе СИ является: [s]=м

В СГС: [s]=см

Примеры решения задач

Пример

Задание. Траектория движения материальной точки изображена на рис. 1. Каков путь, пройденный точкой, чему равно перемещение, если точка двигалась 1-2-3-4.

Решение. Перемещение – кратчайшее расстояние между точками 1 и 4. Следовательно, перемещение точки равно:

$$6 — 2 = 4 (m)$$

Путь – длина траектории. Рассматривая график на рис.1 получаем, что путь материальной точки равен:

$$8 + 4 + 8 = 20 (m)$$

Ответ. Путь равен 20 м, перемещение равно 4 м.

Слишком сложно?

Формула пути не по зубам? Тебе ответит эксперт через 10 минут!

Пример

Задание. Уравнение движения материальной точки в прямоугольной декартовой системе координат представлено функцией: x=-0,2t2 (м) .{5}=5(m)$$

Ответ. s=5м.

Читать дальше: Формула равноускоренного движения.

Перемещение и пройденный путь. Скорость РПД

В прошлый раз мы затронули тему перемещения и скоростей. Остановимся на этом более подробно. Итак, что же такое перемещение тела. Перемещением называется направленный отрезок, проведённый из начального положения тела в его конечное положение.

Поэтому, перемещение — это векторная величина. Действительно, для того, чтобы сказать, как переместилось тело, нам необходимо знать не только расстояние от начальной точки, но и направление, в котором тело переместилось.

Напомним, что положение тела в любой момент времени можно задать с помощью радиус-вектора. Таким образом, перемещение — это изменение радиус-вектора.

Перемещение обозначается латинской буквой , но мы также можем обозначить его как . Необходимо отметить, что перемещение — это не то же самое, что пройденный путь. Пройденный путь — это скалярная величина, которая обозначает расстояние, пройденное телом, в процессе перемещения.

Например, расхаживая по комнате из стороны в сторону, вы можете пройти в общей сложности около ста метров, но ваше перемещение едва ли составит более двух метров. Траектория тела может быть сколь угодно сложной, и именно она будет определять пройденный путь. Перемещение же представляет собой направленный отрезок, соединяющий начальную и конечную точки. А если тело в процессе движения вернулось в исходную точку, то его перемещение будет равно нулю.

Пройденный путь не может быть равен нулю, если тело совершало какое-либо движение.

Возьмём для примера движение тела по ломаной линии. Каждый отрезок ломаной линии имеет длину 1 м. Тогда, пройденный путь будет равен 4 м, а модуль перемещения будет чуть больше двух метров.

Поскольку кратчайшим расстоянием между двумя точками является прямая линия, можно с уверенностью сказать, что модуль перемещения не может быть больше пройденного пути. Не трудно догадаться, что если тело двигается строго по прямой, то модуль перемещения будет равен пройденному пути.

Как вы знаете, довольно важной характеристикой любого движения является скорость. Сегодня мы поговорим о

скорости равномерного прямолинейного движения. Это самый простой вид движения: тело двигается только по прямой и проходит одинаковый путь за равные промежутки времени. Вы уже знакомы с таким видом движения: например, если автомобиль проходит 100 км за час, то за 2 часа он пройдёт 200 км, а за 3 ч — 300 км.

Но, следует понимать, что необходимо задать и направление скорости. На прошлом уроке мы уже убедились, что выбор системы отсчёта имеет решающее значение, а в разных системах отсчета скорости могут быть направлены по-разному.

Скоростью равномерного прямолинейного движения называется величина, равная отношению перемещения к промежутку времени, в течение которого это перемещение произошло:

Скорость является векторной величиной.

Вектор скорости направлен так же, как и вектор перемещения. Это вполне логично: куда тело перемещается, туда и направлена его скорость. Нетрудно догадаться, что если мы поделим модуль перемещения на промежуток времени, за который произошло перемещение, то мы получим модуль скорости:

Эта величина будет говорить нам о том, какое расстояние проходит тело за единицу времени.

Примеры решения задачи.

Задача. Точка является начальной точкой тела с координатами (10;6), а точка является конечной точкой тела с координатами (15;3). Найдите скорость перемещения, если в точке , а в точке , .

Путь при неравномерном движении — материалы для подготовки к ЕГЭ по Физике

 


Автор — профессиональный репетитор, автор учебных пособий для подготовки к ЕГЭ Игорь Вячеславович Яковлев

Сейчас мы будем рассматривать неравномерное движение — то есть движение, при котором абсолютная величина скорости меняется со временем. Оказывается, существует простая геометрическая интерпретация пути, пройденного телом при произвольном движении.

Начнём с равномерного движения. Пусть скорость тела постоянна и равна . Возьмём два момента времени: начальный момент и конечный момент . Длительность рассматриваемого промежутка времени равна .

Очевидно, что за промежуток времени тело проходит путь:

(1)

Давайте построим график зависимости скорости от времени. В данном случае это будет прямая, параллельная оси абсцисс (рис. 1).

Рис. 1. Путь при равномерном движении

 

Нетрудно видеть, что пройденный путь равен площади прямоугольника, расположенного под графиком скорости

. В самом деле, первый множитель в формуле (1) есть вертикальная сторона этого прямоугольника, а второй множитель — его горизонтальная сторона.

Теперь нам предстоит обобщить эту геометрическую интерпретацию на случай неравномерного движения.

Пусть скорость тела зависит от времени, и на рассматриваемом промежутке график скорости выглядит, например, так (рис. 2):

Рис. 2. Неравномерное движение

 

Дальше мы рассуждаем следующим образом.

1. Разобьём наш промежуток времени на небольшие отрезки величиной .

2. Предположим, что на каждом таком отрезке тело движется с постоянной скоростью . То есть, плавное изменение скорости заменим ступенчатой аппроксимацией*: в течение каждого небольшого отрезка времени тело движется равномерно, а затем скорость тела мгновенно и cкачком меняется.
На рис. 3 показаны две ступенчатые аппроксимации. Ширина ступенек на правом рисунке вдвое меньше, чем на левом.

Рис. 3. Ступенчатая аппроксимация

 

Путь, пройденный за время равномерного движения — это площадь прямоугольника, расположенного под ступенькой. Поэтому путь, пройденный за всё время такого «ступенчатого» движения — это сумма площадей всех прямоугольников на графике.

3. Теперь устремляем к нулю. Ясно, что в пределе наша ступенчатая аппроксимация перейдёт в исходный график скорости на рис. 2. Сумма площадей прямоугольников перейдёт в площадь под графиком скорости; следовательно, эта площадь и есть путь, пройденный телом за время от до . (рис. 4

Рис. 4. Путь при неравномерном движении

 

В итоге мы приходим к нужному нам обобщению геометрической интерпретации пути, полученной выше для случая равномерного движения.

Аппроксимация — это приближённая замена достаточно сложного объекта более простой моделью, которую удобнее изучать.

Геометрическая интерпретация пути.Путь, пройденный телом при любом движении, равен площади под графиком скорости на заданном промежутке времени.

Посмотрим, как работает эта геометрическая интерпретация в важном частном случае равноускоренного движения.

Задача. Тело, имеющее скорость в начальный момент , разгоняется с постоянным ускорением . Найти путь, пройденный телом к моменту времени .

Решение. Зависимость скорости от времени в данном случае имеет вид:

(2)

График скорости — прямая, изображённая на рис. 5. Искомый путь есть площадь трапеции, расположенной под графиком скорости.

Рис. 5. Путь при равноускоренном движении

 

Меньшее основание трапеции равно . Большее основание равно . Высота трапеции равна . Поскольку площадь трапеции есть произведение полусуммы оснований на высоту, имеем:

Эту формулу можно переписать в более привычном виде:

Она, разумеется, вам хорошо известна из темы «Равноускоренное движение».

Задача. График скорости тела является полуокружностью диаметра (рис. 6). Максимальная скорость тела равна . Найти путь, пройденный телом за время .

Решение. Как вы знаете, площадь круга радиуса равна . Но в данной задаче необходимо учесть, что радиусы полуокружности имеют разные размерности: горизонтальный радиус есть время , а вертикальный радиус есть скорость .

Поэтому пройденный путь, вычисляемый как площадь полукруга, равен половине произведения на горизонтальный радиус и на вертикальный радиус:

Рис. 6. К задаче

Глава 1. Путь, перемещение, скорость. Движение с постоянной скоростью. Относительность движения

В рамках этой темы необходимо знать ряд простых определений, понимать логику определения скорости и закона сложения скоростей.

Перемещением тела называется вектор, связывающий начальное и конечное положение тела, а пройденным путем — длина траектории. Поэтому величина(или модуль) перемещения — это расстояние от конечной до начальной точки по прямой, а путь — расстояние траектории тела. В задаче 1.1.1 пройденный телом за четверть периода путь — длина четверти окружности , перемещение — (см. рисунок), правильный ответ — 3.

Скорость тела определяется как отношение перемещения тела ко времени , затраченному на это перемещение

(1.1)

Для прямолинейного движения в одном направлении для величины вектора скорости получаем из (1.1)

(1.2)

где — путь, пройденный за время . Если определение (1.1) приводит к одной и той же величине для любого интервала времени , то скорость тела есть величина постоянная, а такое движение называется равномерным (задача 1.1.2 — ответ 4). В этом случае согласно (1.1) и (1.2) перемещение и пройденный путь линейно зависят от времени и . По этой причине линейно зависят от времени и координаты тела в любой системе координат. Поэтому графиком зависимости координат тела от времени для равномерного движения является прямая (задача 1.1.3 — ответ 1). Как следует из (1.1), (1.2), наклон этой прямой определяется скоростью: чем больше скорость, тем «круче» наклонен график зависимости координаты тела от времени к оси времени. Поэтому в задаче 1.1.4 на каждом из интервалов времени — от 0 до 1 с, от 1 до 2 с, от 2 до 3 с и от 3 до 4 с движение тела будет равномерным, а самой большой скорость тела будет в интервале времени от 3 до 4 с, в котором наклон графика максимален (ответ 4).

В задаче 1.1.5 нужно по графику зависимости координаты тела от времени найти его скорость. Это можно сделать так. Перемещение тела внутри каждого из интервалов времени — 0–1, 1–2 и 2–3 с — разность координат тела вначале и в конце этого интервала. Поэтому из графика находим

Таким образом, скорость тела равна 2 м/с внутри интервала времени 1–2 с (ответ 2).

Задача 1.1.6 посвящена размерности скорости. Из определения заключаем, что размерность скорости есть

И, следовательно, размерностью скорости могут быть

(или любые другие отношения единиц расстояний и времени). Для пересчета скорости из одних единиц в другие нужно выразить расстояние и время в требуемых единицах. Например, в задаче 1.1.6 имеем

(правильный ответ — 3).

При движении с постоянной скоростью определения (1.1) или (1.2) могут быть применены к любым этапам движения. Например, в задаче 1.1.7 можно из данных о движении жука вдоль периметра прямоугольника найти его скорость (=14/7=2 см/с), а затем использовать ее для описания движения жука вдоль диагонали (длина которой составляет 5 см): 1=5/2=2,5 с (правильный ответ 2).

Аналогичные соотношения используются в задаче 1.1.8. Рассматривая движение автомобиля на одной трети пути, получаем , где  — расстояние между городами. А на оставшихся двух третях (с учетом трехкратного увеличения скорости) 1. Поэтому полное время движения равно (ответ 1).

В задаче 1.1.9 следует использовать следующее свойство графика зависимости проекции скорости тела на некоторую ось от времени: площадь под этим графиком есть проекция перемещения тела на рассматриваемую ось. Причем площадь под участками графика, лежащими выше оси времени, нужно считать положительной, ниже оси времени — отрицательной. Если же все площадь под всеми участками графика считать положительной, площадь под графиком скорости дает пройденный телом путь. Находя площадь под данным в условии графиком, получаем

(ответ — 4).

Важным физическим законом, знание которого часто проверяется на едином государственном экзамене по физике, является закон сложения скоростей. Этот закон утверждает, что скорости одного и того же тела по отношению к разным системам отсчета связаны соотношением

(1.3)

Здесь и  — скорости тела относительно первой и второй системы отсчета,  — скорость второй системы отсчета относительно первой. Закон сложения скоростей является векторным. Это означает, три вектора , и образуют треугольник векторного сложения, и соотношение между величинами скоростей , и  — такое же, как и между длинами сторон треугольника. Углы этого треугольника равны углам между направлениями скоростей , и .

Примеры треугольников сложения скоростей приведены на рисунке, причем на среднем и правом рисунке приведены примеры «треугольников» скоростей в случаях, когда скорость тела в системе 2 и скорость системы 2 относительно системы 1 направлены одинаково (средний рисунок) и противоположно (правый рисунок). Из этих рисунков следует, что скалярное соотношение, аналогичное (1.3) для величин скоростей , справедливо только, если векторы и направлены одинаково (средний рисунок). Если же векторы и направлены противоположно, для значений скоростей справедливо соотношение (или наоборот , если  — правый рисунок. Из этих рассуждений ясно, что поскольку в задаче 1.1.10 векторы скорости пассажира относительно поезда и поезда относительно земли направлены одинаково, скорость пассажира относительно земли равна (правильный ответ — 2). В задаче 1.2.1 ситуация обратная — вектор скорости первой машины относительно земли и второй машины относительно земли направлены противоположно. Поэтому , направлен вектор на север — правильный ответ 4.

В задаче 1.2.2 эти идеи применяются к движению лодки по и против течения. Из закона сложения скоростей заключаем, что при движении лодки по течению ее скорость относительно земли равна , при движении против течения — ( — скорость лодки в стоячей воде,  — скорость течения). Отсюда находим, что при движении лодки по течению, ее скорость относительно земли 15 км/ч, а при движении против течения — 5 км/ч. Поэтому время движения между городами и по течению втрое больше времени движения лодки между этими городами против течения (ответ — 2).

Все следующие задачи этой главы являются более сложными, поскольку в них рассматривается движение не одного, а двух тел, а закон сложения скоростей используется в случаях, когда векторы скоростей не направлены вдоль одной прямой. В задаче 1.2.3 встреча тел происходит в такой точке, что расстояния, пройденные первым и вторым телом, отличаются втрое (так как в три раза отличаются скорости тел). Поэтому при выходе из точки тела встретятся в такой точке , что длины дуг отличаются в три раза. Следовательно, угол  — прямой, и длина отрезка равна . (ответ 4).

Если два тела, начав движение одновременно, движутся навстречу друг другу (задача 1.2.4), то время встречи тел можно найти следующим образом. Так как тела двигались до встречи одинаковое время, они прошли расстояния и , сумма которых равна первоначальному расстоянию между телами . Поэтому (ответ 2). Отметим, что данные в условии задачи ответы 3 и 4 имеют неправильную размерность — 1/с и потому могут быть отброшены сразу. Задача 1.2.5 решается с помощью таких соображений: время движения первого пешехода между городами , второго — , встречи пешеходов (см. предыдущую задачу). Отсюда

Сокращая в этой формуле величину , получаем

или ч (правильный ответ — 1).

В задаче 1.2.6 начальное и конечное положения вагона и человека показаны на правой и левой частях рисунка.

Отсюда заключаем, что разность перемещений вагона и человека равна длине вагона . Поэтому время, через которое провожающий окажется около конца вагона, определяется из соотношения . Из этой формулы находится время, а затем и расстояние, пройденное провожающим (ответ 1). Отметим, что ответы 3 и 4 могли быть отброшены сразу, поскольку не описывают случай одинаковых скоростей. Действительно, при одинаковых скоростях вагон никогда не обгонит провожающего, и расстояние, пройденное при «обгоне» провожающим, должно обратиться в бесконечность. Другими словами, ответ должен содержать нуль в знаменателе при .

Задача 1.2.7 посвящена вычислению средней скорости движения на некотором пути, которая определяется как отношение этого пути к затраченному времени. Если расстояние между городами и равно , то полное время движения между городами складывается из времен, затраченных на первую и вторую половины пути

Отсюда находим км/ч (правильный ответ — 3).

В задачах 1.2.8–1.2.9 закон сложения скоростей рассматривается в ситуациях, когда векторы , и направлены не вдоль одной прямой. В этом случае необходимо использовать закон сложения скоростей в векторной форме (1.3). Когда человек в поезде идет перпендикулярно направлению его движения (задача 1.2.8), треугольник сложения скоростей (1.3) имеет вид, показанный на рисунке.

Здесь  — вектор скорости поезда относительно земли,  — вектор скорости человека относительно поезда, который по условию направлен перпендикулярно вектору . Поэтому согласно закону сложения скоростей вектор скорости человека относительно земли представляет собой гипотенузу прямоугольного треугольника, катетами которого являются векторы и (см. рисунок). Следовательно, величину скорости человека относительно земли можно найти по теореме Пифагора (ответ 3).

Задачи 1.2.9. и 1.2.10 удобнее решать, переходя из той системы отсчета, в которой задача поставлена (в системе отсчета, связанной с землей) в некоторую другую систему, в которой рассматриваемое явление является более простым. При переправе через реку (задача 1.2.9) скорость лодки относительно земли зависит от траектории — на траекториях, направленных под острыми углами к течению, скорость лодки больше, чем на траекториях, на которых угол между скоростью лодки и скоростью течения — тупой. Поэтому время переправы по самой короткой траектории (перпендикулярной берегам) не является минимальным. Траекторию с минимальным временем переправы легко найти в системе отсчета, связанной с водой. В этой системе отсчета вода покоится, и, следовательно, минимальное время переправы достигается на такой траектории, на которой вектор скорости лодки относительно воды перпендикулярен берегам реки. Поэтому вектор скорости лодки относительно земли на этой траектории наклонен под углом к течению (см. рисунок). Под таким углом к берегу и расположена траектория, на переправу по которой лодка затрачивает минимальное время (правильный ответ — 1).

В задаче 1.2.10 рассматривается движение трех тел. В системе отсчета, связанной с землей ответ неочевиден. Быстрый катер дольше уплывет от лодки, но будет двигаться быстрее и при обратном движении, медленный — наоборот. Однако если перейти в систему отсчета, связанную с водой, решение очень несложно. В этой системе отсчета плот покоится, каждый катер при движении от плота и к плоту движется с одинаковой скоростью. Поэтому каждый катер вернется к плоту через то же самое время после разворота, в течение которого он двигался от плота. Следовательно, катера вернутся одновременно (ответ 3).

Путь при равноускоренном движении | Физика

Зная среднюю скорость и время движения, можно найти пройденный путь:

Подставляя в эту формулу выражение (3.3), мы найдем путь, пройденный при равноускоренном движении из состояния покоя:

Если же мы подставим в формулу (4.1) выражение (3.4), то получим путь, пройденный при торможении:

В последние две формулы входят скорости v0 и v. Их можно найти по формулам (3.1) и (3.2). Подставляя выражение (3.1) в формулу (4.2), а выражение (3.2) — в формулу (4.3), получим

Полученная формула справедлива как для равноускоренного движения из состояния покоя, так и для движения с уменьшающейся скоростью, когда тело в конце пути останавливается. В обоих этих случаях пройденный путь пропорционален квадрату времени движения (а не просто времени, как это было в случае равномерного движения). Первым, кто установил эту закономерность, был Г. Галилей.

В таблице 2 даны основные формулы, описывающие равноускоренное прямолинейное движении.

Своей книги, в которой излагалась теория равноускоренного движения (наряду со многими другими его открытиями), Галилею увидеть не довелось. Когда она была издана, 74-летний ученый был уже слепым. Галилей очень тяжело переживал потерю зрения. «Вы можете себе представить, писал он,— как я горюю, когда я сознаю, что это небо, этот мир и Вселенная, которые моими наблюдениями и ясными доказательствами расширены в сто и в тысячу раз по сравнению с тем, какими их считали люди науки во все минувшие столетия, теперь для меня так уменьшились и сократились».

За пять лет до этого Галилей был подвергнут суду инквизиции. Его взгляды на устройство мира (а он придерживался системы Коперника, в которой центральное место занимало Солнце, а не Земля) уже давно не нравились служителям церкви. Еще в 1614 г. доминиканский священник Каччини объявил Галилея еретиком, а математику — изобретением дьявола. А в 1616 г. инквизиция официально заявила, что «учение, приписываемое Копернику, что Земля движется вокруг Солнца, Солнце же стоит в центре Вселенной, не двигаясь с востока на запад, противно Священному писанию, а потому его не можно ни защищать, ни принимать за истину». Книга Коперника с изложением его системы мира была запрещена, а Галилея предупредили, что если «он не успокоится, то его подвергнут заключению в тюрьму».

Но Галилей «не успокоился». «В мире нет большей ненависти,— писал ученый,— чем у невежества к знанию». И в 1632 г. выходит его знаменитая книга «Диалог о двух главнейших системах мира — птолемеевой и коперниковой», в которой он привел многочисленные аргументы в пользу системы Коперника. Однако продать удалось всего лишь 500 экземпляров этого сочинения, так как уже через несколько месяцев по распоряжению Папы Римского издатель книги получил приказ приостановить продажу этого труда.

Осенью того же года Галилей получает предписание инквизиции явиться в Рим, и через некоторое время больного 69-летнего ученого на носилках доставляют в столицу. Здесь, в тюрьме инквизиции, Галилея заставляют отречься от своих взглядов на устройство мира, и 22 июня 1633 г. в римском монастыре Минервы Галилей зачитывает и подписывает заранее приготовленный текст отречения:

«Я, Галилео Галилей, сын покойного Винченцо Галилея из Флоренции, 70 лет от роду, доставленный лично на суд и коленоприклоненный перед Вашими Преосвященствами, высокопреподобными господами кардиналами, генеральными инквизиторами против ереси во всем христианском мире, имея перед собой священное Евангелие и возлагая на него руки, клянусь, что я всегда верил, верую ныне и с Божией помощью буду веровать впредь во все то, что святая католическая и апостольская римская церковь признает, определяет и проповедует».

Согласно решению суда, книга Галилея была запрещена, а сам он был приговорен к тюремному заключению на неопределенный срок. Однако Папа Римский помиловал Галилея и заменил заключение в тюрьме изгнанием. Галилей переезжает в Арчетри и здесь, находясь под домашним арестом, пишет книгу «Беседы и математические доказательства, касающиеся двух новых отраслей науки, относящихся к Механике и Местному движению». В 1636 г. рукопись книги была переправлена в Голландию, где и была издана в 1638 г. Этой книгой Галилей подводил итог своим многолетним физическим исследованиям.

В том же году Галилей полностью ослеп. Рассказывая о постигшем великого ученого несчастье, Вивиани (ученик Галилея) писал: «Случились у него тяжкие истечения из глаз, так что спустя несколько месяцев совсем остался он без глаз — да, говорю я, без своих глаз, которые за краткое время увидели в этом мире более, чем все человеческие глаза за все ушедшие столетия смогли увидеть и наблюсти».

Посетивший Галилея флорентийский инквизитор в своем письме в Рим сообщил, что нашел его в очень тяжелом состоянии. На основании этого письма Папа Римский разрешил Галилею вернуться в родной дом во Флоренции. Здесь ему сразу же вручили предписание: «Под страхом пожизненного заключения в истинную тюрьму и отлучения от церкви не выходить в город и ни с кем, кто бы это ни был, не говорить о проклятом мнении насчет двоякого движения Земли».

У себя дома Галилей пробыл недолго. Через несколько месяцев ему снова было приказано приехать в Арчетри. Жить ему оставалось около четырех лет. 8 января 1642 г. в четыре часа ночи Галилей умер.

1. Чем отличается равноускоренное движение от равномерного? 2. Чем отличается формула пути при равноускоренном движении от формулы пути при равномерном движении? 3. Что вы знаете о жизни и творчестве Г. Галилея? В каком году он родился?

Урок физики по теме «Расчет пути и времени движения». 7-й класс

Ключевые слова: скорость, время, расстояние, расчет пути, расчет времени движения

(Презентация, слайд 1)

Цели урока: научить учеников вычислять путь, пройденный телом при равномерном движении и время движения; повышать интерес к физике, как науке об окружающем нас мире (слайд 2)

ХОД УРОКА

I. Повторение (слайд 3)

  • Что называют движением?
  • Какие виды движения вы знаете?
  • Что понимают под скоростью?
  • Как вычислить скорость?
  • В каких величинах измеряется скорость?

Проверка упр.4(2,4).

– Ребята, а какие вы знаете пословицы о скорости?

Ответы учеников:

–  Долго рассуждай, да скоро делай!
–  Семеро одного не ждут.
–  Большой меньшого не дожидается.
–  Под гору вскачь, а на гору хоть плачь.
–  Тише едешь, дальше будешь. 
–  Делу – время, потехе – час.

II. Изучение

1. Прямолинейное равномерное движение (слайд 4). [4]
2. Так как V = S/t, то S = V · t, а также t = S/V   (слайд 5).
3. Схема для запоминания формул расчета скорости, времени, пути:

Рис.1

4. Графики скорости и пути равномерного движения: (слайд 7)

Скорость – величина векторная (слайд 6) [4], поэтому она характеризуется не только модулем, но и направлением. В зависимости от выбранного положительным направления скорость может быть постоянной по величине, но положительной (если движение происходит по выбранному как положительное направлению) или отрицательной по знаку (если движение происходит противоположно выбранному направлению)

А) Графики скорости равномерного движения:

Рис. 2

Б) График пути равномерного движения:

Рис. 3

5. Вычисление пути и времени неравномерного движения: (слайд 8)

Vср = S / t, где S – весь пройденный телом путь, а t – все время движения.

Формулы для вычисления пути и времени будут иметь тот же вид: S = Vср · t, а также t = S / Vср, но в расчет берут среднюю скорость движения.

Если в течение рассматриваемого промежутка времени направление движения не менялось, то средняя скорость перемещения равняется средней скорости в пути, в том случае, когда движение происходило в сторону возрастающих расстояний, или равняется отрицательно взятой средней скорости в пути, если движение совершалось в сторону убывающих расстояний.

III. Закрепление

1. Обсуждение вопросов к п.16.

2. Решение задач: (слайды 9-11)

1)  Упр. 5(2):

На велосипеде без особого напряжения можно ехать со скоростью 3 м/c. На какое расстояние можно уехать за 1,5 часа?

Дано
V = 3 м/с
t = 1,5 ч
Найти: S
СИ

5400с 

Решение
S = V·t   
S = 3м/c· 5400 с = 16200 м = 16,2 км
Ответ: 16,2 км

2)  Упр. 5(3):

На рисунке 38 показан график зависимости пути равномерного движения от времени. Оs – ось пройденных путей, Оt – ось времени. По этому графику найдите, чему равен путь, пройденный телом за 2 часа. Затем рассчитайте скорость тела.

Дано
t = 2ч
S = 200 км
Найти: V
Решение
V = S/t
V = 200 км/2 ч = 100 км/ч
Ответ: 100 км/ч

3)  Упр. 5(4):

График зависимости скорости равномерного движения тела от времени представлен на рисунке 39. По этому графику определите скорость движения тела. Рассчитайте путь, который пройдет тело за 2ч, 4ч.

Дано
V=8 м/с 
t1 = 2ч
t2 = 4ч
Найти: S1 и S2
СИ

7200с
14400с

Решение
S = V·t  
S1 = 8м/с 7200с = 57500м = 57,6 км.
S2 = 8м/с 14400с = 115200м = 115,2 км
Ответ: 57,6км и 115,2 км

IV. Самостоятельное решение задач с последующим обсуждением полученных результатов (слайды 12-14).

а) Это интересно:

1. Самый быстрый зверь на Земле = гепард. Эта стройная пятнистая кошка на длиннющих ногах развивает скорость 110 км/ч. Но бежит недалеко. Если сразу не догонит жертву, тут же возвращается в засаду. Какое расстояние пробегает гепард за 5 секунд? (153 м).   [2]

2. Несмотря на маленькие размеры колибри способны пролететь значительные расстояния. Например, рубиноголовая колибри весной и осенью перелетают Мексиканский залив, преодолевая без остановки примерно 900 км со скоростью 40 км/ч. При порхающем полете крылья колибри движутся со скоростью 3000-4800 взмахов в минуту. Определите время перелета колибри через Мексиканский залив.     (81000с = 22ч.30мин).   [2]

3. Самыми быстрокрылыми считаются ласточки и стрижи. Скорость их полета до 150 км/ч. Какое расстояние пролетают эти птицы за минуту? (2500м). [2]

б)   Смешинки (слайды16-17):

1.  Двигаясь равномерно и нигде не задерживаясь, мелкий отличник Олег добрался из дома до школы за 7 мин.  Его соседка крупная троечница Оля тоже двигалась равномерно, также не задерживалась и добралась до школы за 57 мин. Чем отличался стремительный и целеустремленный бег в школу отличника Олега Хлопушина от тихой и печальной поступи троечницы Оли Ромашкиной?    [3]

2.  Печальный дядя Боря несет из магазина домой авоську с мелкими куриными яйцами. Через равные промежутки времени яйца из авоськи выпадают на землю и разбиваются. Можно ли сказать, что печальный дядя Боря движется равномерно, если коты, бегущие за ним, находят яйца на равных расстояниях одно за другим? [3]

V. Подведение итогов урока, выставление оценок

VI. Рефлексия

– Что нового вы сегодня узнали на уроке?
– Что вам больше всего понравилось?

VII. Домашнее задание (слайд 18)

П.16, упр.5(4,5), придумать и решить две задачи на расчет пути и времени равномерного движения. Постарайтесь сделать их оригинальными.

Использованные ресурсы:

  1. А.В.Перышкин. Физика – 7ласс «Дрофа», 2006год.
  2. А.М.Семке. «Физика. Занимательные материалы к урокам. 7 класс.» Москва, «НЦ ЭРФС» 2006г
  3. Г.Остер. Физика задачник, Москва, «Росмэн»,1995г
  4. Физика. Библиотека наглядных пособий. 7-11 кл. ООО «1С», 2004-2011, диск.
  5. Интернет ресурсы.

Как рассчитывать траектории | Sciencing

Обновлено 28 декабря 2020 г.

Кевин Бек

Движение снаряда относится к движению частицы, которой сообщается начальная скорость, но впоследствии на нее не действуют никакие силы, кроме силы тяжести.

Сюда входят задачи, в которых частица бросается под углом от 0 до 90 градусов к горизонтали, причем горизонталь обычно является землей. Для удобства предполагается, что эти снаряды движутся в плоскости ( x, y ), причем x представляют горизонтальное смещение, а y — вертикальное смещение.

Путь, пройденный снарядом, называется его траекторией . (Обратите внимание, что общее звено в словах «снаряд» и «траектория» — это слог «-дъект», латинское слово, означающее «бросить». Выбросить кого-либо — это буквально выбросить его.) Точка происхождения снаряда в задачах в котором вам нужно рассчитать траекторию, обычно предполагается, что она равна (0, 0) для простоты, если не указано иное.

Траектория снаряда представляет собой параболу (или, по крайней мере, отслеживает часть параболы), если частица запускается таким образом, что имеет ненулевую составляющую горизонтального движения и отсутствует сопротивление воздуха, влияющее на частицу.

Кинематические уравнения

Переменными, представляющими интерес в движении частицы, являются ее координаты положения x и y , ее скорость v и ее ускорение a , все по отношению к данному истекшему времени t с момента начала проблемы (когда частица запущена или выпущена). Обратите внимание, что отсутствие массы (m) означает, что гравитация на Земле действует независимо от этой величины.

Отметим также, что эти уравнения игнорируют роль сопротивления воздуха, которое создает силу сопротивления, противодействующую движению в реальных земных условиях.Этот фактор вводится на курсах механики более высокого уровня.

Переменные с нижним индексом «0» относятся к значению этой величины в момент времени t = 0 и являются константами; часто это значение равно 0 благодаря выбранной системе координат, и уравнение становится намного проще. В этих задачах ускорение рассматривается как постоянное (оно в направлении оси y и равно — g, или –9,8 м / с 2 , ускорение, вызванное силой тяжести у поверхности Земли).2-2g (y-y_0)

Примеры движения снаряда

Ключом к решению задач, включающих расчет траектории, является знание того, что горизонтальную (x) и вертикальную (y) компоненты движения можно анализировать отдельно, так как показано выше, и их вклад в общее движение аккуратно суммирован в конце задачи.

Проблемы с движением снаряда считаются проблемами свободного падения, потому что, как бы все ни выглядело сразу после времени t = 0, единственной силой, действующей на движущийся объект, является гравитация.

  • Имейте в виду, что поскольку сила тяжести направлена ​​вниз, а это считается отрицательным направлением оси y, значение ускорения в этих уравнениях и задачах равно -g.

Расчет траектории

1. Самые быстрые питчеры в бейсболе могут бросить мяч со скоростью чуть более 100 миль в час или 45 м / с. Если мяч подброшен вертикально вверх с этой скоростью, насколько высоко он поднимется и сколько времени потребуется, чтобы вернуться в точку, в которой он был выпущен?

Здесь v y0 = 45 м / с, — g = –9. 2 — (2) (9.8) (y — 0) = 2,025 — 19,6y \ подразумевает y = 103,3 \ text {m}

Уравнение v y = v 0y — gt показывает, что время t, необходимое для этого, равно ( 45 / 9,8) = 4,6 секунды. Чтобы получить общее время, прибавьте это значение ко времени, которое требуется мячу, чтобы свободно упасть в исходную точку. Это определяется как y = y 0 + v 0y t — (1/2) gt 2 , где теперь, поскольку мяч все еще находится в данный момент прежде, чем он начнет резко падать, v 0y = 0.2 \ подразумевает t = 4,59 \ text {s}

Таким образом, общее время составляет 4,59 + 4,59 = 9,18 секунды. Возможно, удивительный результат, когда каждый «этап» путешествия, вверх и вниз, занимал одно и то же время, подчеркивает тот факт, что гравитация является единственной силой, действующей здесь.

2. Уравнение дальности: Когда снаряд запускается со скоростью v 0 и углом θ от горизонтали, он имеет начальную горизонтальную и вертикальную составляющие скорости v 0x = v 0 (cos θ) и v 0y = v 0 (sin θ). 2 \ sin {2 \ theta}} {g}

(Последний шаг следует из тригонометрического тождества 2 sinθ ⋅ cosθ = sin 2θ.2} {g}

Смещение | Физика

Чтобы описать движение объекта, вы должны сначала уметь описать его положение — где он находится в любой конкретный момент времени. Точнее, нужно указать его положение относительно удобной системы отсчета. Земля часто используется в качестве системы отсчета, и мы часто описываем положение объекта по отношению к неподвижным объектам в этой системе отсчета. Например, запуск ракеты можно описать с точки зрения положения ракеты по отношению к Земле в целом, а положение профессора можно описать с точки зрения ее положения по отношению к соседней белой доске.(См. Рис. 2.) В других случаях мы используем системы отсчета, которые не являются стационарными, но находятся в движении относительно Земли. Например, чтобы описать положение человека в самолете, мы используем самолет, а не Землю в качестве системы отсчета. (См. Рисунок 3.)

Если объект движется относительно системы отсчета (например, если профессор движется вправо относительно белой доски или пассажир движется к задней части самолета), то положение объекта изменяется. Это изменение положения известно как смещение .Слово «смещение» означает, что объект переместился или был перемещен.

Рабочий объем

Смещение — это изменение на позиции объекта:

Δ x = x f x o,

, где Δ x — смещение, x f — конечное положение, а x 0 — начальное положение.

В этом тексте заглавная греческая буква Δ (дельта) всегда означает «изменение» любой следующей за ней величины; таким образом, Δ x означает изменение позиции .Всегда вычисляйте смещение, вычитая начальное положение x 0 из конечного положения x f .

Обратите внимание, что единицей СИ для смещения является метр (м) (см. Физические величины и единицы), но иногда используются километры, мили, футы и другие единицы длины. Имейте в виду, что когда в задаче используются единицы, отличные от счетчика, вам может потребоваться преобразовать их в метры, чтобы завершить расчет.

Рис. 2. Профессор ходит влево и вправо во время лекции.Ее положение относительно Земли обозначено x. Смещение профессора относительно Земли представлено стрелкой, указывающей вправо.

Обратите внимание, что смещение имеет направление, а также величину. Смещение профессора составляет 2,0 м вправо, а смещение пассажира авиакомпании — 4,0 м назад. В одномерном движении направление может быть указано со знаком плюс или минус. Когда вы начинаете решать проблему, вы должны выбрать, какое направление является положительным (обычно это будет вправо или вверх, но вы можете выбрать положительное как любое направление).Начальное положение профессора: x 0 = 1,5 м, а конечное положение: x f = 3,5 м. Таким образом, ее водоизмещение составляет

.

Δ x = x f x o = 3,5 м — 1,5 м = +2,0 м

В этой системе координат движение вправо положительно, а движение влево — отрицательно. Точно так же исходное положение пассажира самолета — x 0 = 6,0 м, а его конечное положение — x f = 2.0 м, значит, его водоизмещение

Δ x = x f x o = 2,0 м — 6,0 м = −4,0 м

Его смещение отрицательно, потому что его движение направлено к задней части плоскости или в отрицательном направлении x в нашей системе координат.

Расстояние

Хотя смещение описывается с точки зрения направления, расстояние — нет. Расстояние определяется как величина или размер смещения между двумя положениями .Обратите внимание, что расстояние между двумя позициями не равно расстоянию, пройденному между ними. Пройденное расстояние составляет общая длина пути, пройденного между двумя позициями . Расстояние не имеет направления и, следовательно, знака. Например, расстояние, которое проходит профессор, составляет 2,0 м. Расстояние, которое проходит пассажир самолета — 4,0 м.

Предупреждение о заблуждении: пройденное расстояние в зависимости от величины смещения

Важно отметить, что пройденное расстояние , однако, может быть больше, чем величина смещения (под величиной мы подразумеваем просто размер смещения без учета его направления; то есть просто число с единица измерения).Например, профессор может много раз ходить взад и вперед, возможно, пройти 150 м во время лекции, но все же закончить только 2,0 м справа от своей начальной точки. В этом случае ее смещение будет +2,0 м, величина ее смещения будет 2,0 м, но пройденное ею расстояние составит 150 м. В кинематике мы почти всегда имеем дело со смещением и величиной смещения и почти никогда не с пройденным расстоянием. Один из способов подумать об этом — предположить, что вы отметили начало и конец движения.Смещение — это просто разница в положении двух меток и не зависит от пути, пройденного между двумя метками. Однако пройденное расстояние — это общая длина пути, пройденного между двумя отметками.

Проверьте свое понимание

Велосипедист едет на 3 км на запад, затем разворачивается и едет на 2 км на восток. а) Каково ее смещение? б) Какое расстояние она проезжает? в) Какова величина ее перемещения?

Рисунок 4.

Решения

(a) Смещение всадника составляет Δ x = x f x o = -1 км. (Смещение отрицательное, потому что мы считаем восток положительным, а запад — отрицательным.)

(b) Пройденное расстояние составляет 3 км + 2 км = 5 км.

(c) Величина смещения составляет 1 км.

Сводка раздела

  • Кинематика — это исследование движения без учета его причин.В этой главе оно ограничено движением по прямой линии, называемым одномерным движением.
  • Смещение — это изменение положения объекта.
  • В символах смещение Δ x определено как

Δ x = x f x o,

, где x o — начальное положение, а x f — конечное положение. В этом тексте греческая буква Δ (дельта) всегда означает «изменение» любой следующей за ней величины.Единицей измерения перемещения в системе СИ является метр (м). Смещение имеет направление, а также величину.

  • Когда вы начинаете проблему, назначьте, какое направление будет положительным.
  • Расстояние — это величина смещения между двумя положениями.
  • Пройденное расстояние — это общая длина пути между двумя позициями.

Концептуальные вопросы

1. Приведите пример, в котором есть четкие различия между пройденным расстоянием, смещением и величиной смещения.Определите каждое количество в вашем примере.

2. При каких обстоятельствах пройденное расстояние равно величине смещения? Каков единственный случай, когда величина смещения и смещения точно совпадают?

3. Бактерии перемещаются вперед и назад, используя свои жгутики (структуры, похожие на маленькие хвосты). Наблюдались скорости до 50 мкм / с (50 c 10 -6 м / с). Общее расстояние, которое проходит бактерия, велико для ее размера, а перемещение невелико.Почему это?

Скорость, ускорение и сила | Безграничная физика

Угол вращения и угловая скорость

Угол поворота — это мера того, как далеко вращается объект, а угловая скорость — это скорость его вращения.

Цели обучения

Выразите взаимосвязь между углом поворота и расстоянием

Основные выводы

Ключевые моменты
  • Когда объект вращается вокруг оси, точки на краю объекта перемещаются по дугам.
  • Угол, выходящий за пределы этих дуг, называется углом поворота и обычно обозначается символом theta .
  • Мера того, насколько быстро объект вращается относительно времени, называется угловой скоростью. Обычно он представлен греческим символом омега . Как и его аналог линейной скорости, это вектор.
Ключевые термины
  • радиан : угол, образуемый в центре окружности дугой той же длины, что и радиус окружности.

Угол вращения и угловая скорость

Когда объект вращается вокруг оси, как в случае с шиной автомобиля или записью на поворотной платформе, движение можно описать двумя способами. Точка на краю вращающегося объекта будет иметь некоторую скорость и будет перенесена по дуге на вращающемся объекте. Точка будет перемещаться на расстояние [latex] \ Delta \ text {S} [/ latex], но часто удобнее говорить о степени поворота объекта. Величина поворота объекта называется углом поворота и может измеряться в градусах или радианах.Поскольку угол поворота связан с расстоянием [latex] \ Delta \ text {S} [/ latex] и с радиусом [latex] \ text {r} [/ latex] уравнением [latex] \ Delta \ theta = \ frac {\ Delta \ text {S}} {\ text {R}} [/ latex], обычно удобнее использовать радианы.

Угол θ и длина дуги s : Радиус круга поворачивается на угол [latex] \ Delta \ theta [/ latex]. Длина дуги [латекс] \ Delta \ text {s} [/ latex] указывается на окружности.

Скорость вращения объекта определяется угловой скоростью, которая представляет собой скорость изменения угла поворота во времени.Хотя сам угол не является векторной величиной, угловая скорость — это вектор. Направление вектора угловой скорости перпендикулярно плоскости вращения в направлении, которое обычно задается правилом правой руки. Угловое ускорение дает скорость изменения угловой скорости. Угол, угловая скорость и угловое ускорение очень полезны при описании вращательного движения объекта.

Направление угловой скорости : Угловая скорость описывает скорость вращения и ориентацию мгновенной оси, вокруг которой происходит вращение.Направление угловой скорости будет вдоль оси вращения. В этом случае (вращение против часовой стрелки) вектор направлен вверх.

Когда ось вращения перпендикулярна вектору положения, угловую скорость можно вычислить, взяв линейную скорость [latex] \ text {v} [/ latex] точки на краю вращающегося объекта и разделив на радиус. Это даст угловую скорость, обычно обозначаемую [latex] \ omega [/ latex], в радианах в секунду.

Угловая скорость : Муха на краю вращающегося объекта фиксирует постоянную скорость [latex] \ text {v} [/ latex]. Объект вращается с угловой скоростью, равной [latex] \ frac {\ text {v}} {\ text {r}} [/ latex].

Центробежное ускорение

Центростремительное ускорение — это постоянное изменение скорости, необходимое объекту для поддержания круговой траектории.

Цели обучения

Выразите центростремительное ускорение через скорость вращения

Основные выводы

Ключевые моменты
  • Для того, чтобы объект сохранял круговое движение, он должен постоянно менять направление.
  • Поскольку скорость является вектором, изменения направления представляют собой изменения скорости.
  • Изменение скорости называется ускорением. Изменение скорости из-за кругового движения известно как центростремительное ускорение.
  • Центростремительное ускорение может быть рассчитано путем деления квадрата линейной скорости на радиус круга, по которому движется объект.
Ключевые термины
  • ускорение : величина, на которую увеличивается скорость или скорость (и, следовательно, скалярная величина или векторная величина).
  • круговое движение : движение таким образом, что выбранная траектория представляет собой круговую траекторию.
  • скорость : векторная величина, которая обозначает скорость изменения положения относительно времени или скорость с направленным компонентом.

Обзор

Как упоминалось в предыдущих разделах по кинематике, любое изменение скорости определяется ускорением. Часто изменения скорости являются изменениями по величине. Когда объект ускоряется или замедляется, это изменение скорости объекта.Изменения в величине скорости соответствуют нашему интуитивному и повседневному использованию термина «ускорение». Однако, поскольку скорость является вектором, у нее также есть направление. Следовательно, любое изменение направления движения объекта также должно сопровождаться ускорением.

Равномерное круговое движение означает, что объект движется по круговой траектории с постоянной скоростью. Поскольку скорость постоянна, обычно не думают, что объект ускоряется. Однако направление постоянно меняется, когда объект пересекает круг.Таким образом, говорят, что он ускоряется. Это ускорение можно почувствовать, катаясь на американских горках. Даже если скорость постоянная, быстрый поворот вызовет у гонщика чувство силы. Это ощущение ускорения.

Центростремительное ускорение : Краткий обзор центростремительного ускорения для школьников-физиков.

Расчет центростремительного ускорения

Для расчета центростремительного ускорения объекта, совершающего равномерное круговое движение, необходимо иметь скорость, с которой движется объект, и радиус круга, вокруг которого происходит движение.2 \ text {r} [/ latex]

, где омега — это скорость вращения, заданная [latex] \ frac {\ text {v}} {\ text {r}} [/ latex].

Центростремительное ускорение : Когда объект движется по окружности, направление вектора скорости постоянно меняется.

Центростремительная сила

Сила, которая вызывает движение по криволинейной траектории, называется центростремительной силой (равномерное круговое движение является примером центростремительной силы).

Цели обучения

Выразите уравнения для центростремительной силы и ускорения

Основные выводы

Ключевые моменты
  • Когда объект находится в равномерном круговом движении, он постоянно меняет направление и, следовательно, ускоряется.Это угловое ускорение.
  • Сила, действующая на объект при равномерном круговом движении (называемая центростремительной силой), действует на объект из центра круга.
Ключевые термины
  • центростремительный : Направлен или движется к центру.
  • угловая скорость : векторная величина, описывающая объект в круговом движении; его величина равна скорости частицы, а направление перпендикулярно плоскости ее кругового движения.

Сила, вызывающая движение по криволинейной траектории, называется центростремительной силой. Равномерное круговое движение является примером действия центростремительной силы. Это можно увидеть на орбите спутников вокруг Земли, натяжении веревки в игре с тросом, в петле-петле на американских горках или в ведре, вращающемся вокруг тела.

Обзор центростремительной силы : Краткий обзор центростремительной силы.

Ранее мы узнали, что любое изменение скорости — это ускорение.По мере того, как объект движется по круговой траектории, он постоянно меняет направление и, следовательно, ускоряется, вызывая постоянное воздействие на объект силы. Эта центростремительная сила действует по направлению к центру кривизны, по направлению к оси вращения. Поскольку объект движется перпендикулярно силе, путь, по которому он движется, является круговым. Именно эта сила удерживает мяч от выпадения из ведра, если вы непрерывно раскачиваете его по кругу.

Центростремительная сила : Когда объект движется по круговой траектории с постоянной скоростью, он испытывает центростремительную силу, ускоряющую его к центру.2 [/ латекс]

3.4 Движение с постоянным ускорением — Университетская физика, том 1

Цели обучения

К концу этого раздела вы сможете:

  • Определите, какие уравнения движения следует использовать для решения неизвестных.
  • Используйте соответствующие уравнения движения для решения задачи преследования двух тел.

Вы можете догадаться, что чем больше ускорение, скажем, автомобиля, удаляющегося от знака «Стоп», тем больше смещение автомобиля за данный момент времени.Но мы не разработали конкретное уравнение, которое связывает ускорение и смещение. В этом разделе мы рассмотрим некоторые удобные уравнения кинематических отношений, начиная с определений смещения, скорости и ускорения. Сначала мы исследуем движение одного объекта, называемого движением одного тела. Затем мы исследуем движение двух объектов, называемых задачами преследования двух тел .

Обозначение

Во-первых, сделаем несколько упрощений в обозначениях.Принятие начального времени равным нулю, как если бы время измерялось секундомером, является большим упрощением. Поскольку прошедшее время

, принимая

означает, что

, последнее время на секундомере. Когда начальное время принимается равным нулю, мы используем индекс 0 для обозначения начальных значений положения и скорости. То есть

— начальная позиция и

— начальная скорость .Мы не ставим индексы на окончательные значения. То есть t — это конечный момент времени , x — конечная позиция , а v — конечная скорость . Это дает более простое выражение для прошедшего времени:

.

. Это также упрощает выражение для смещения x , которое теперь составляет

. Кроме того, это упрощает выражение для изменения скорости, которое теперь составляет

.

. Подводя итог, используя упрощенные обозначения, с начальным временем, принятым равным нулю,

, где нижний индекс 0 обозначает начальное значение, а отсутствие нижнего индекса означает конечное значение в любом рассматриваемом движении.

Теперь мы делаем важное предположение, что ускорение постоянно . Это предположение позволяет нам избегать использования расчетов для определения мгновенного ускорения. Поскольку ускорение постоянно, среднее и мгновенное ускорения равны, то есть

Таким образом, мы можем использовать символ a для ускорения в любое время. Предположение, что ускорение является постоянным, не серьезно ограничивает ситуации, которые мы можем изучить, и не ухудшает точность нашего лечения.Во-первых, ускорение равно постоянно в большом количестве ситуаций. Кроме того, во многих других ситуациях мы можем точно описать движение, приняв постоянное ускорение, равное среднему ускорению для этого движения. Наконец, для движения, во время которого ускорение резко меняется, например, когда автомобиль разгоняется до максимальной скорости, а затем тормозит до остановки, движение можно рассматривать в отдельных частях, каждая из которых имеет собственное постоянное ускорение.

Смещение и положение от скорости

Чтобы получить наши первые два уравнения, мы начнем с определения средней скорости:

Замена

упрощенным обозначением

и

дает

Решение для x дает нам

, где средняя скорость

Уравнение

отражает тот факт, что при постоянном ускорении v — это просто среднее значение начальной и конечной скоростей.(Рисунок) графически иллюстрирует эту концепцию. В части (а) рисунка ускорение является постоянным, а скорость увеличивается с постоянной скоростью. Средняя скорость на 1-часовом интервале от 40 км / ч до 80 км / ч составляет 60 км / ч:

В части (b) ускорение не является постоянным. В течение 1-часового интервала скорость ближе к 80 км / ч, чем к 40 км / ч. Таким образом, средняя скорость больше, чем в части (а).

Рисунок 3.18 (a) График зависимости скорости от времени с постоянным ускорением, показывающий начальную и конечную скорости

.Средняя скорость

. (б) График зависимости скорости от времени с изменением ускорения со временем. Средняя скорость не указана в

.

, но больше 60 км / ч.

Решение окончательной скорости по ускорению и времени

Мы можем вывести еще одно полезное уравнение, манипулируя определением ускорения:

Замена

упрощенным обозначением

и

дает нам

Решение для v дает

Пример

Расчет конечной скорости

Самолет приземляется с начальной скоростью 70.0 м / с, а затем замедляется со скоростью 1,50 м / с 2 в течение 40,0 с. Какова его конечная скорость?

Стратегия

Во-первых, мы идентифицируем известные:

.

Во-вторых, мы идентифицируем неизвестное; в данном случае это конечная скорость

.

Наконец, мы определяем, какое уравнение использовать. Для этого мы выясняем, какое кинематическое уравнение дает неизвестное в терминах известных. Мы рассчитываем окончательную скорость, используя (Рисунок),

.

Решение

[Показать-ответ q = ”287818 ″] Показать ответ [/ Показать-ответ]
[hidden-answer a =” 287818 ″] Подставить известные значения и решить:

(рисунок) — это эскиз, на котором показаны векторы ускорения и скорости. [/ Hidden-answer]

Рисунок 3.19 Самолет приземляется с начальной скоростью 70,0 м / с и замедляется до конечной скорости 10,0 м / с, прежде чем направиться к терминалу. Обратите внимание, что ускорение отрицательное, потому что его направление противоположно его скорости, которая положительна.
Значение

Конечная скорость намного меньше начальной скорости, требуемой при замедлении, но все же положительная (см. Рисунок). С реактивными двигателями обратная тяга может поддерживаться достаточно долго, чтобы остановить самолет и начать движение назад, на что указывает отрицательная конечная скорость, но в данном случае это не так.

Уравнение

не только помогает при решении задач.

дает нам представление о взаимосвязи между скоростью, ускорением и временем.Мы видим, например, что

  • Конечная скорость зависит от того, насколько велико ускорение и как долго оно длится
  • Если ускорение равно нулю, то конечная скорость равна начальной скорости ( v = v 0 ), как и ожидалось (другими словами, скорость постоянна)
  • Если a отрицательное, то конечная скорость меньше начальной скорости

Все эти наблюдения соответствуют нашей интуиции. Обратите внимание, что всегда полезно исследовать основные уравнения в свете нашей интуиции и опыта, чтобы убедиться, что они действительно точно описывают природу.

Решение для конечного положения с постоянным ускорением

Мы можем объединить предыдущие уравнения, чтобы найти третье уравнение, которое позволяет нам вычислить окончательное положение объекта, испытывающего постоянное ускорение. Начнем с

Добавление

в каждую сторону этого уравнения и деление на 2 дает

С

для постоянного разгона имеем

Теперь подставляем это выражение вместо

в уравнение перемещения,

, давая

Пример

Расчет смещения ускоряющегося объекта

Драгстеры могут развивать среднее ускорение 26.0 м / с 2 . Предположим, драгстер ускоряется из состояния покоя в течение 5,56 с (рисунок). Как далеко он пролетит за это время?

Рисунок 3.20 Пилот Top Fuel американской армии Тони «Сержант» Шумахер начинает гонку с контролируемым выгоранием. (Источник: подполковник Уильям Термонд. Фотография предоставлена ​​армией США.)
Стратегия

Сначала нарисуем эскиз (рисунок). Нас просят найти смещение, которое составляет x , если мы возьмем

равняется нулю.(Подумайте о

как стартовая линия гонки. Он может быть где угодно, но мы называем его нулем и измеряем все остальные положения относительно него.) Мы можем использовать уравнение

, когда мы идентифицируем

,

, и т. из постановки задачи.

Рис. 3.21 Эскиз разгоняющегося драгстера.
Решение

[показать-ответ q = ”9 ″] Показать ответ [/ показать-ответ]
[скрытый-ответ a =” 9 ″] Во-первых, нам нужно определить известные.Запуск из состояния покоя означает, что

, a равно 26,0 м / с2, а t равно 5,56 с.
Во-вторых, мы подставляем известные значения в уравнение, чтобы найти неизвестное:

Поскольку начальное положение и скорость равны нулю, это уравнение упрощается до

Подстановка идентифицированных значений a и t дает

[/ hidden-answer]

Значение

Если мы переведем 402 м в мили, мы обнаружим, что пройденное расстояние очень близко к четверти мили, стандартному расстоянию для дрэг-рейсинга.Итак, наш ответ разумный. Это впечатляющий водоизмещение всего за 5,56 с, но первоклассные драгстеры могут преодолеть четверть мили даже за меньшее время. Если бы драгстеру была присвоена начальная скорость, это добавило бы еще один член в уравнение расстояния. Если в уравнении использовать те же ускорение и время, пройденное расстояние будет намного больше.

Что еще мы можем узнать, исследуя уравнение

Мы видим следующие отношения:

  • Смещение зависит от квадрата истекшего времени, когда ускорение не равно нулю.На (Рисунок) драгстер преодолевает только одну четверть общего расстояния за первую половину прошедшего времени.
  • Если ускорение равно нулю, то начальная скорость равна средней скорости

    и

Расчет конечной скорости по расстоянию и ускорению

Четвертое полезное уравнение может быть получено путем другой алгебраической обработки предыдущих уравнений. Если мы решим

за т , получаем

Подставляя это и

в

, получаем

Пример

Расчет конечной скорости

Рассчитайте окончательную скорость драгстера (рисунок) без использования информации о времени.

Стратегия

Уравнение

идеально подходит для этой задачи, поскольку он связывает скорости, ускорение и смещение и не требует информации о времени.

Решение

[show-answer q = ”350935 ″] Показать ответ [/ show-answer]
[hidden-answer a =” 350935 ″] Сначала мы идентифицируем известные значения. Мы знаем, что v0 = 0, поскольку драгстер стартует из состояния покоя. Мы также знаем, что x — x0 = 402 м (это был ответ на (Рисунок)).Среднее ускорение составило a = 26,0 м / с2.

ПЕРЕРЫВ Во-вторых, мы подставляем известные в уравнение

и решите относительно v:

ПЕРЕРЫВ

Таким образом, ПЕРЕРЫВ

[/ hidden-answer]

Значение

Скорость 145 м / с составляет около 522 км / ч или около 324 миль / ч, но даже эта головокружительная скорость не достигает рекорда для четверти мили. Также обратите внимание, что квадратный корень имеет два значения; мы взяли положительное значение, чтобы указать скорость в том же направлении, что и ускорение.

Исследование уравнения

может дать дополнительную информацию об общих отношениях между физическими величинами:

  • Конечная скорость зависит от величины ускорения и расстояния, на котором оно действует.
  • При фиксированном ускорении автомобиль, который едет вдвое быстрее, не просто останавливается на удвоенном расстоянии. Чтобы остановиться, нужно гораздо дальше. (Вот почему у нас есть зоны с пониженной скоростью возле школ.)

Объединение уравнений

В следующих примерах мы продолжаем исследовать одномерное движение, но в ситуациях, требующих немного большего количества алгебраических манипуляций.Примеры также дают представление о методах решения проблем. Следующее примечание предназначено для облегчения поиска необходимых уравнений. Имейте в виду, что эти уравнения не являются независимыми. Во многих ситуациях у нас есть два неизвестных, и нам нужно два уравнения из набора для решения для неизвестных. Для решения данной ситуации нам нужно столько уравнений, сколько неизвестных.

Сводка кинематических уравнений (константа a )

Прежде чем мы перейдем к примерам, давайте более внимательно рассмотрим некоторые уравнения, чтобы увидеть поведение ускорения при экстремальных значениях.Переставляя (рисунок), получаем

Из этого мы видим, что в течение конечного времени, если разница между начальной и конечной скоростями мала, ускорение невелико, приближаясь к нулю в пределе, когда начальная и конечная скорости равны. Напротив, в лимите

для конечной разницы между начальной и конечной скоростями ускорение становится бесконечным.

Аналогично, переставив (рисунок), мы можем выразить ускорение в терминах скоростей и смещения:

Таким образом, при конечной разнице между начальной и конечной скоростями ускорение становится бесконечным, в пределе смещение приближается к нулю.Ускорение приближается к нулю в пределе, разница в начальной и конечной скоростях приближается к нулю для конечного смещения.

Пример

Как далеко уезжает машина?

На сухом бетоне автомобиль может замедляться со скоростью 7,00 м / с 2 , тогда как на мокром бетоне он может замедляться только со скоростью 5,00 м / с 2 . Найдите расстояния, необходимые для остановки автомобиля, движущегося со скоростью 30,0 м / с (около 110 км / ч) по (а) сухому бетону и (б) мокрому бетону. (c) Повторите оба вычисления и найдите смещение от точки, где водитель видит, что светофор становится красным, принимая во внимание время его реакции, равное 0.500 с, чтобы нажать на педаль тормоза.

Стратегия

Для начала нам нужно нарисовать эскиз (рисунок). Чтобы определить, какие уравнения лучше всего использовать, нам нужно перечислить все известные значения и точно определить, что нам нужно решить.

Рисунок 3.22 Образец эскиза для визуализации замедления и тормозного пути автомобиля.
Решение
  1. Во-первых, нам нужно определить известные и то, что мы хотим решить. Мы знаем, что v 0 = 30.0 м / с, v = 0 и a = -7,00 м / с 2 ( a отрицательно, потому что оно находится в направлении, противоположном скорости). Возьмем x 0 равным нулю. Ищем перемещение

    или x x 0 . Во-вторых, мы определяем уравнение, которое поможет нам решить проблему. Лучшее уравнение для использования —

    .

    Это уравнение лучше всего, потому что оно включает только одно неизвестное, x .Мы знаем значения всех других переменных в этом уравнении. (Другие уравнения позволили бы нам решить для x , но они требуют, чтобы мы знали время остановки, t , которое мы не знаем. Мы могли бы их использовать, но это потребовало бы дополнительных вычислений.)

    В-третьих, мы изменим уравнение, чтобы найти x :

    и подставьте известные значения:

    Таким образом,

  2. Эта часть может быть решена точно так же, как (а).Единственное отличие состоит в том, что ускорение составляет −5,00 м / с 2 . Результат

  3. [Показать-ответ q = ”175639 ″] Показать ответ [/ Показать-ответ]
    [hidden-answer a =” 175639 ″] Когда водитель реагирует, тормозной путь такой же, как в пунктах (а) и ( б) для сухого и влажного бетона. Итак, чтобы ответить на этот вопрос, нам нужно вычислить, как далеко проехал автомобиль за время реакции, а затем добавить это время ко времени остановки. Разумно предположить, что скорость остается постоянной в течение времени реакции водителя.Для этого мы снова определяем известные и то, что мы хотим решить. Мы знаем это

    ,

    и

    . Берем

    равняется нулю. Ищем

    . Во-вторых, как и раньше, мы определяем лучшее уравнение для использования. В данном случае

    работает хорошо, потому что единственное неизвестное значение — это x, которое мы и хотим найти.В-третьих, мы подставляем известные, чтобы решить уравнение:

    Это означает, что автомобиль движется на 15,0 м, пока водитель реагирует, в результате чего общее смещение в двух случаях с сухим и мокрым бетоном на 15,0 м больше, чем при мгновенной реакции. Наконец, мы добавляем смещение во время реакции к смещению при торможении ((Рисунок)),

    и найти (а) равным 64,3 м + 15,0 м = 79,3 м в сухом состоянии и (б) равным 90,0 м + 15,0 м = 105 м во влажном состоянии.[/ hidden-answer]

Рисунок 3.23 Расстояние, необходимое для остановки автомобиля, сильно варьируется в зависимости от дорожных условий и времени реакции водителя. Здесь показаны значения тормозного пути для сухого и мокрого покрытия, рассчитанные в этом примере для автомобиля, изначально движущегося со скоростью 30,0 м / с. Также показано общее расстояние, пройденное от точки, когда водитель впервые видит красный свет, при условии, что время реакции составляет 0,500 с.
Значение

Смещения, найденные в этом примере, кажутся разумными для остановки быстро движущегося автомобиля.Остановка автомобиля на мокром асфальте должна длиться дольше, чем на сухом. Интересно, что время реакции значительно увеличивает смещения, но более важен общий подход к решению проблем. Мы идентифицируем известные и определяемые величины, а затем находим соответствующее уравнение. Если существует более одного неизвестного, нам нужно столько независимых уравнений, сколько неизвестных необходимо решить. Часто есть несколько способов решить проблему. Фактически, различные части этого примера могут быть решены другими методами, но представленные здесь решения являются самыми короткими.

Пример

Время расчета

Предположим, что автомобиль выезжает на автомагистраль на съезде длиной 200 м. Если его начальная скорость составляет 10,0 м / с, а ускорение составляет 2,00 м / с 2 , сколько времени потребуется автомобилю, чтобы преодолеть 200 м по рампе? (Такая информация может быть полезна транспортному инженеру.)

Стратегия

Сначала рисуем эскиз (рисунок). Нам предлагается решить за время т . Как и раньше, мы идентифицируем известные величины, чтобы выбрать удобное физическое соотношение (то есть уравнение с одним неизвестным, t .)

Рис. 3.24 Эскиз автомобиля, ускоряющегося на съезде с автострады.
Решение

[show-answer q = ”712029 ″] Показать ответ [/ show-answer]
[hidden-answer a =” 712029 ″] Опять же, мы определяем известные нам и то, что мы хотим решить. Мы знаем, что

, и x = 200 м.

Нам нужно решить для t. Уравнение

работает лучше всего, потому что единственное неизвестное в уравнении — это переменная t, для которой нам нужно решить.Из этого понимания мы видим, что когда мы вводим известные значения в уравнение, мы получаем квадратное уравнение.

Нам нужно изменить уравнение, чтобы найти t, а затем подставить известные значения в уравнение:

Затем мы упрощаем уравнение. Единицы измерения отменяются, потому что они есть в каждом члене. Мы можем получить единицы секунд для отмены, взяв t = t s, где t — величина времени, а s — единица измерения. Остается

Затем мы используем формулу корней квадратного уравнения, чтобы найти t,

, что дает два решения: t = 10.0 и t = -20,0. Отрицательное значение времени неразумно, так как это будет означать, что событие произошло за 20 секунд до начала движения. Мы можем отказаться от этого решения. Таким образом,

[/ hidden-answer]

Значение

Всякий раз, когда уравнение содержит неизвестный квадрат, есть два решения. В некоторых проблемах имеют смысл оба решения; в других разумно только одно решение. Ответ 10,0 с кажется разумным для типичной автострады на съезде.

Проверьте свое понимание

Пилотируемая ракета ускоряется со скоростью 20 м / с. 2 во время пуска.Сколько времени нужно, чтобы ракета достигла скорости 400 м / с?

[показывать-ответ q = ”fs-id1168329484424 ″] Показать решение [/ раскрыть-ответ]

[скрытый-ответ a = ”fs-id1168329484424 ″]

Чтобы ответить на этот вопрос, выберите уравнение, которое позволяет нам решить для времени t , учитывая только a , v 0 и v :

Перегруппировать для решения для т :

[/ hidden-answer]

Пример

Ускорение космического корабля

Космический корабль покинул орбиту Земли и направляется к Луне.Разгоняется со скоростью 20 м / с 2 за 2 мин и преодолевает расстояние в 1000 км. Каковы начальная и конечная скорости космического корабля?

Стратегия

Нас просят найти начальную и конечную скорости космического корабля. Глядя на кинематические уравнения, мы видим, что одно уравнение не дает ответа. Мы должны использовать одно кинематическое уравнение для решения одной из скоростей и подставить его в другое кинематическое уравнение, чтобы получить вторую скорость. Таким образом, мы решаем два кинематических уравнения одновременно.

Решение

[show-answer q = ”835228 ″] Показать ответ [/ show-answer]
[hidden-answer a =” 835228 ″] Сначала мы решаем для

с использованием

Затем подставляем

в

, чтобы найти окончательную скорость:

[/ hidden-answer]

Значение

Есть шесть переменных: смещение, время, скорость и ускорение, которые описывают движение в одном измерении.Начальные условия данной задачи могут быть множеством комбинаций этих переменных. Из-за такого разнообразия решения могут быть нелегкими, например простой заменой в одно из уравнений. Этот пример показывает, что решения кинематики могут потребовать решения двух одновременных кинематических уравнений.

Освоив основы кинематики, мы можем перейти ко многим другим интересным примерам и приложениям. В процессе разработки кинематики мы также увидели общий подход к решению проблем, который дает как правильные ответы, так и понимание физических взаимоотношений.Следующий уровень сложности в наших задачах кинематики связан с движением двух взаимосвязанных тел, называемых задачами преследования двух тел .

Задачи преследования двух тел

До этого момента мы рассматривали примеры движения с участием одного тела. Даже для задачи с двумя автомобилями и тормозным путем на мокрой и сухой дороге мы разделили эту задачу на две отдельные задачи, чтобы найти ответы. В задаче о преследовании двух тел движения объектов связаны, то есть искомая неизвестная зависит от движения обоих объектов.Чтобы решить эти проблемы, мы пишем уравнения движения для каждого объекта, а затем решаем их одновременно, чтобы найти неизвестное. Это проиллюстрировано на (Рисунок).

Рис. 3.25 Сценарий преследования с двумя телами, при котором автомобиль 2 имеет постоянную скорость, а автомобиль 1 отстает с постоянным ускорением. Автомобиль 1 догонит автомобиль 2 позже.

Время и расстояние, необходимое для того, чтобы автомобиль 1 догнал автомобиль 2, зависят от начального расстояния, на которое автомобиль 1 находится от автомобиля 2, а также от скорости обоих автомобилей и ускорения автомобиля 1.Чтобы найти эти неизвестные, необходимо решить кинематические уравнения, описывающие движение обеих машин.

Рассмотрим следующий пример.

Пример

Гепард ловит газель

Гепард прячется за кустом. Гепард замечает пробегающую мимо газель со скоростью 10 м / с. В тот момент, когда газель проходит мимо гепарда, гепард из состояния покоя ускоряется со скоростью 4 м / с 2 , чтобы поймать газель. а) Сколько времени требуется гепарду, чтобы поймать газель? б) Что такое смещение газели и гепарда?

Стратегия

Мы используем систему уравнений для постоянного ускорения, чтобы решить эту проблему.Поскольку есть два движущихся объекта, у нас есть отдельные уравнения движения, описывающие каждое животное. Но то, что связывает уравнения, — это общий параметр, который имеет одинаковое значение для каждого животного. Если мы внимательно посмотрим на проблему, становится ясно, что общим параметром для каждого животного является их положение x , позднее t . Поскольку они оба начинаются с

, их смещения такие же, в более позднее время t , когда гепард догоняет газель.Если мы выберем уравнение движения, которое решает смещение для каждого животного, мы можем затем установить уравнения, равные друг другу, и решить для неизвестного, то есть времени.

Решение
  1. [показать-ответ q = ”699945 ″] Показать ответ [/ показать-ответ]
    [скрытый-ответ a =” 699945 ″] Уравнение для газели: газель имеет постоянную скорость, которая является ее средней скоростью, поскольку это не ускоряется. Поэтому мы используем (рисунок) с

    :

    Уравнение для гепарда: гепард ускоряется из состояния покоя, поэтому мы используем (рисунок) с

    .

    и

    :

    Теперь у нас есть уравнение движения для каждого животного с общим параметром, который можно исключить, чтобы найти решение.В этом случае мы решаем для t:

    Газель имеет постоянную скорость 10 м / с, что является ее средней скоростью. Ускорение гепарда составляет 4 м / с2. Оценивая t, время, за которое гепард достигает газели, получаем

    [/ hidden-answer]

  2. [показать-ответ q = ”316146 ″] Показать ответ [/ раскрыть-ответ]
    [скрытый-ответ a =” 316146 ″] Чтобы получить смещение, мы используем уравнение движения гепарда или газели, поскольку они оба должны дать одинаковый ответ.Смещение гепарда:

    Водоизмещение газели:

    Мы видим, что оба смещения равны, как и ожидалось. [/ Hidden-answer]

Значение

Важно анализировать движение каждого объекта и использовать соответствующие кинематические уравнения для описания отдельного движения. Также важно иметь хорошую визуальную перспективу задачи преследования двух тел, чтобы увидеть общий параметр, который связывает движение обоих объектов.

Проверьте свое понимание

Велосипед имеет постоянную скорость 10 м / с. Человек стартует с отдыха и бежит догонять велосипед за 30 с. Какое ускорение у человека?

[show-answer q = ”fs-id1168326827870 ″] Показать решение [/ show-answer]

[скрытый-ответ a = ”fs-id1168326827870 ″]

.
[/ hidden-answer]

Резюме

  • При анализе одномерного движения с постоянным ускорением определите известные величины и выберите соответствующие уравнения для решения неизвестных.Для решения неизвестных необходимы одно или два кинематических уравнения, в зависимости от известных и неизвестных величин.
  • Задачи двухчастичного преследования всегда требуют одновременного решения двух уравнений относительно неизвестных.

Концептуальные вопросы

При анализе движения отдельного объекта, какое количество известных физических переменных необходимо для решения неизвестных величин с использованием кинематических уравнений?

Укажите два сценария кинематики одного объекта, в которых три известные величины требуют решения двух кинематических уравнений для неизвестных.

[show-answer q = ”fs-id1168326

5 ″] Показать решение [/ show-answer]

[скрытый-ответ a = ”fs-id1168326

5 ″]

Если ускорение, время и перемещение являются известными, а начальная и конечная скорости являются неизвестными, то два кинематических уравнения должны решаться одновременно. Также, если конечная скорость, время и смещение являются известными, тогда необходимо решить два кинематических уравнения для начальной скорости и ускорения.

[/ hidden-answer]

Проблемы

Частица движется по прямой с постоянной скоростью 30 м / с.Каково его смещение между t = 0 и t = 5,0 с?

[показывать-ответ q = ”fs-id1168326

4 ″] Показать решение [/ показывать-ответ]

[скрытый-ответ a = ”fs-id1168326

4 ″]

150 кв.м

[/ hidden-answer]

Частица движется по прямой с начальной скоростью 30 м / с и постоянным ускорением 30 м / с 2 . Если на

и

, каково положение частицы при t = 5 с?

Частица движется по прямой с начальной скоростью 30 м / с и постоянным ускорением 30 м / с 2 .(а) Какое у него водоизмещение при т = 5 с? б) Какова его скорость в это же время?

[показывать-ответ q = ”fs-id11683262 ″] Показать решение [/ показывать-ответ]

[скрытый-ответ a = ”fs-id11683262 ″]

а. 525 м;

г.

[/ hidden-answer]

(a) Нарисуйте график зависимости скорости от времени, соответствующий графику перемещения от времени, представленному на следующем рисунке. (b) Определите время или времена ( t a , t b , t c и т. д.), при которой мгновенная скорость имеет наибольшее положительное значение. (c) В какое время он равен нулю? (г) В какое время он отрицательный?


[show-answer q = ”966010 ″] Показать ответ [/ show-answer]
[hidden-answer a =” 966010 ″] [/ hidden-answer]

(a) Нарисуйте график зависимости ускорения от времени, соответствующий графику зависимости скорости от времени, представленному на следующем рисунке. (b) Определите время или времена ( t a , t b , t c и т. д.), при котором ускорение имеет наибольшее положительное значение. (c) В какое время он равен нулю? (г) В какое время он отрицательный?


[раскрыть-ответ q = ”

6 ″] Показать ответ [/ раскрыть-ответ]

[hidden-answer a = ”

6 ″]

а.

г. Ускорение имеет наибольшее положительное значение на

.

г. Ускорение нулевое на

г. Ускорение отрицательное на

[/ hidden-answer]

Частица имеет постоянное ускорение 6.0 м / с 2 . (а) Если его начальная скорость составляет 2,0 м / с, в какое время его смещение составляет 5,0 м? б) Какова его скорость в то время?

При t = 10 с частица движется слева направо со скоростью 5,0 м / с. При t = 20 с частица движется справа налево со скоростью 8,0 м / с. Предполагая, что ускорение частицы постоянное, определите (а) ее ускорение, (б) ее начальную скорость и (в) момент, когда ее скорость равна нулю.

[show-answer q = ”fs-id1168327148264 ″] Показать решение [/ show-answer]

[скрытый-ответ a = ”fs-id1168327148264 ″]

а.

;
г.

;

г.

[/ hidden-answer]

Хорошо брошенный мяч попадает в рукавицу с хорошей набивкой. Если ускорение мяча

и 1,85 мс

проходит с момента первого прикосновения мяча к рукавице до остановки. Какова начальная скорость мяча?

Пуля в ружье ускоряется от камеры выстрела до конца ствола со средней скоростью

.

для

.Какова его начальная скорость (то есть конечная скорость)?

[show-answer q = ”fs-id1168329484717 ″] Показать решение [/ show-answer]

[скрытый-ответ a = ”fs-id1168329484717 ″]

[/ hidden-answer]

(a) Пригородный легкорельсовый поезд ускоряется со скоростью 1,35 м / с 2 . Сколько времени нужно, чтобы достичь максимальной скорости 80,0 км / ч, начиная с состояния покоя? (b) Этот же поезд обычно замедляется со скоростью 1,65 м / с 2 .Сколько времени нужно, чтобы остановиться с максимальной скорости? (c) В аварийных ситуациях поезд может замедляться быстрее, останавливаясь на скорости 80,0 км / ч за 8,30 с. Каково его аварийное ускорение в метрах на секунду в квадрате?

При выезде на автостраду автомобиль ускоряется из состояния покоя со скоростью 2,04 м / с. 2 за 12,0 с. (а) Нарисуйте набросок ситуации. (б) Перечислите известных в этой проблеме. (c) Как далеко проехал автомобиль за эти 12,0 с? Чтобы решить эту часть, сначала определите неизвестное, а затем укажите, как вы выбрали соответствующее уравнение для его решения.После выбора уравнения покажите свои шаги в поиске неизвестного, проверьте свои единицы и обсудите, является ли ответ разумным. (d) Какова конечная скорость автомобиля? Решите для этого неизвестного таким же образом, как в (c), явно показывая все шаги.

[show-answer q = ”fs-id1168327145386 ″] Показать решение [/ show-answer]

[скрытый-ответ a = ”fs-id1168327145386 ″]

а.

г. Знает:

и

;

г.

, ответ кажется разумным на высоте около 172,8 м; d.

[/ hidden-answer]

Необоснованные результаты В конце забега бегун замедляется со скорости 9,00 м / с со скоростью 2,00 м / с 2 . а) Как далеко она продвинется в следующие 5,00 с? б) Какова ее конечная скорость? (c) Оцените результат. Имеет ли это смысл?

Кровь ускоряется из состояния покоя до 30,0 см / с на расстоянии 1.80 см от левого желудочка сердца. (а) Сделайте набросок ситуации. (б) Перечислите известных в этой проблеме. (c) Сколько времени длится ускорение? Чтобы решить эту часть, сначала определите неизвестное, а затем обсудите, как вы выбрали соответствующее уравнение для его решения. После выбора уравнения покажите свои шаги в решении неизвестного, проверяя свои единицы. (г) Является ли ответ разумным по сравнению со временем биения сердца?

[показывать-ответ q = ”fs-id1168329325655 ″] Показать решение [/ показывать-ответ]

[скрытый-ответ a = ”fs-id1168329325655 ″]

а.

г. Знает:

;

г.

;

г. да

[/ hidden-answer]

Во время удара по шлепку хоккеист разгоняет шайбу со скорости 8,00 м / с до 40,0 м / с в том же направлении. Если на этот снимок уйдет

, на каком расстоянии разгоняется шайба?

Мощный мотоцикл может разогнаться с места до 26.8 м / с (100 км / ч) всего за 3,90 с. а) Каково его среднее ускорение? б) Как далеко он пролетит за это время?

[показывать-ответ q = ”fs-id1168329293321 ″] Показать решение [/ показывать-ответ]

[скрытый-ответ a = ”fs-id1168329293321 ″]

а. 6,87 с 2 ; б.

[/ hidden-answer]

Грузовые поезда могут развивать только относительно небольшие ускорения. (а) Какова конечная скорость грузового поезда, который ускоряется со скоростью

?

для 8.00 мин, начиная с начальной скорости 4,00 м / с? (б) Если поезд может замедлиться со скоростью

, сколько времени потребуется, чтобы остановиться на этой скорости? (c) Как далеко он продвинется в каждом случае?

Снаряд фейерверка ускоряется из состояния покоя до скорости 65,0 м / с на расстоянии 0,250 м. (а) Рассчитайте ускорение. б) Как долго длилось ускорение?

[показывать-ответ q = ”fs-id1168326954581 ″] Показать решение [/ показывать-ответ]

[скрытый-ответ a = ”fs-id1168326954581 ″]

а.

;
г.

[/ hidden-answer]

Лебедь на озере поднимается в воздух, взмахивая крыльями и бегая по воде. (a) Если лебедь должен достичь скорости 6,00 м / с для взлета и ускоряется из состояния покоя со средней скоростью

, как далеко он пролетит, прежде чем взлетит? б) Сколько времени это займет?

Мозг дятла особенно защищен от сильных ускорений связками внутри черепа, похожими на сухожилия.Во время клевания дерева голова дятла останавливается с начальной скорости 0,600 м / с на расстоянии всего 2,00 мм. (a) Найдите ускорение в метрах в секунду в квадрате и кратно g , где g = 9,80 м / с 2 . (b) Рассчитайте время остановки. (c) Сухожилия, удерживающие мозг, растягиваются, делая его тормозной путь 4,50 мм (больше, чем голова и, следовательно, меньше ускорение мозга). Каково ускорение мозга, кратное g ?

[показывать-ответ q = ”fs-id1168326955141 ″] Показать решение [/ показывать-ответ]

[скрытый-ответ a = ”fs-id1168326955141 ″]

а.

г.

;

г.

[/ hidden-answer]

Неосторожный футболист сталкивается со стойкой ворот с мягкой подкладкой при беге со скоростью 7,50 м / с и полностью останавливается, сжав подушку и свое тело на 0,350 м. а) Каково его ускорение? б) Как долго длится столкновение?

Посылка выпадает из грузового самолета и приземляется в лесу. Если предположить, что скорость посылки при ударе составляет 54 м / с (123 мили в час), то каково ее ускорение? Предположим, деревья и снег останавливают его на расстоянии 3.0 мин.

[show-answer q = ”fs-id1168326

9 ″] Показать решение [/ show-answer]

[скрытый-ответ a = ”fs-id1168326

9 ″]

Знает:

. Нам нужны a , поэтому мы можем использовать это уравнение:

.
[/ hidden-answer]

Скоростной поезд проходит через станцию. Он входит с начальной скоростью 22,0 м / с и замедляется со скоростью

.

как проходит.Длина станции 210,0 м. а) Как быстро он движется, когда нос покидает станцию? б) Какова длина носа поезда на станции? (c) Если длина поезда 130 м, какова скорость конца поезда, когда он уезжает? (d) Когда поезд отправляется со станции?

Неоправданные результаты Драгстеры могут развить максимальную скорость 145,0 м / с всего за 4,45 с. (а) Рассчитайте среднее ускорение для такого драгстера. (b) Найдите конечную скорость этого драгстера, начиная с состояния покоя и ускоряясь со скоростью, найденной в (a) для 402.0 м (четверть мили) без использования информации о времени. (c) Почему конечная скорость больше той, которая используется для определения среднего ускорения? ( Подсказка : подумайте, справедливо ли предположение о постоянном ускорении для драгстера. Если нет, обсудите, будет ли ускорение больше в начале или в конце пробега и как это повлияет на конечную скорость.)

[показать-ответ q = ”fs-id1168329316432 ″] Показать решение [/ раскрыть-ответ]

[скрытый-ответ a = ”fs-id1168329316432 ″]

а.

;
г.

;

г.

, потому что предположение о постоянном ускорении недействительно для драгстера. Драгстер переключает передачи и будет иметь большее ускорение на первой передаче, чем на второй, чем на третьей, и так далее. Вначале ускорение будет максимальным, поэтому на

не будет.

за последние несколько метров, но существенно меньше, и конечная скорость будет меньше

.

.

[/ hidden-answer]

Глоссарий

задача преследования двух тел
задача кинематики, в которой неизвестные вычисляются путем одновременного решения кинематических уравнений для двух движущихся объектов

2.2 Скорость и скорость — физика

Цели обучения секции

К концу этого раздела вы сможете делать следующее:

  • Рассчитать среднюю скорость объекта
  • Связать смещение и среднюю скорость

Поддержка учителей

Поддержка учителей

Цели обучения в этом разделе помогут вашим ученикам овладеть следующими стандартами:

  • (4) Научные концепции.Учащийся знает и применяет законы движения в самых разных ситуациях. Ожидается, что студент:
    • (B) описывают и анализируют движение в одном измерении, используя уравнения с понятиями расстояния, смещения, скорости, средней скорости, мгновенной скорости и ускорения.

Кроме того, руководство лаборатории физики средней школы рассматривает содержание этого раздела лаборатории под названием «Положение и скорость объекта», а также следующие стандарты:

  • (4) Научные концепции.Учащийся знает и применяет законы движения в самых разных ситуациях. Ожидается, что студент:
    • (В) описывать и анализировать движение в одном измерении, используя уравнения с понятиями расстояния, смещения, скорости, средней скорости, мгновенной скорости и ускорения.

Раздел Основные термины

средняя скорость средняя скорость мгновенная скорость
мгновенная скорость скорость скорость

Поддержка учителя

Поддержка учителя

В этом разделе учащиеся применяют то, что они узнали о расстоянии и смещении, к понятиям скорости и скорости.

[BL] [OL] Прежде чем студенты прочитают раздел, попросите их привести примеры того, как они слышали слово «скорость». Затем спросите их, слышали ли они слово «скорость». Объясните, что в повседневной жизни эти слова часто используются как синонимы, но их научные определения различаются. Скажите студентам, что они узнают об этих различиях по мере чтения раздела.

[AL] Объясните учащимся, что скорость, как и смещение, является векторной величиной. Попросите их поразмышлять о том, чем скорость отличается от скорости.После того, как они поделятся своими идеями, задайте вопросы, которые углубят их мыслительный процесс, например: Почему вы так думаете? Какой пример? Как можно применить эти термины к движению, которое вы видите каждый день?

Скорость

Движение — это не только расстояние и смещение. Такие вопросы, как: «Сколько времени занимает пешая гонка?» и «Какая была скорость бегуна?» невозможно ответить без понимания других концепций. В этом разделе мы рассмотрим время, скорость и скорость, чтобы расширить наше понимание движения.

Описание того, насколько быстро или медленно движется объект, — это его скорость. Скорость — это скорость, с которой объект меняет свое местоположение. Как и расстояние, скорость является скаляром, потому что у нее есть величина, но не направление. Поскольку скорость — это показатель, она зависит от временного интервала движения. Вы можете рассчитать прошедшее время или изменение времени ΔtΔt движения как разницу между временем окончания и временем начала

Единицей времени в СИ является секунда (с), а единицей измерения скорости в системе СИ являются метры в секунду (м / с), но иногда километры в час (км / ч), мили в час (миль / ч) или другие единицы измерения. скорость используются.

Когда вы описываете скорость объекта, вы часто описываете среднее значение за период времени. Средняя скорость, v avg , представляет собой пройденное расстояние, разделенное на время, в течение которого происходит движение.

vavg = distancetimevavg = distancetime

Вы, конечно, можете изменить уравнение, чтобы найти расстояние или время

время = distancevavg.time = distancevavg. distance = vavg × timedistance = vavg × time

Предположим, например, что автомобиль проезжает 150 километров за 3 секунды.2 часа. Его средняя скорость за поездку —

. vavg = расстояние-время = 150 км3,2 ч = 47 км / ч. vavg = расстояние-время = 150 км3,2 ч = 47 км / ч.

Скорость автомобиля, вероятно, увеличится и уменьшится во много раз за 3,2 часа поездки. Однако его скорость в определенный момент времени — это его мгновенная скорость. Спидометр автомобиля показывает его мгновенную скорость.

Teacher Support

Teacher Support

[OL] [AL] Предупредите учащихся, что средняя скорость не всегда равна средней начальной и конечной скорости объекта.Например, предположим, что автомобиль проезжает 100 км. Первые 50 км он движется со скоростью 30 км / ч, а вторые 50 км — со скоростью 60 км / ч. Его средняя скорость будет составлять расстояние / (временной интервал) = (100 км) / [(50 км) / (30 км / ч) + (50 км) / (60 км / ч)] = 40 км / ч. Если бы автомобиль на этих скоростях проехал равное время на 30 и 60 км, а не на равные расстояния, его средняя скорость составила бы 45 км / ч.

[BL] [OL] Предупредите учащихся, что термины «скорость», «средняя скорость» и «мгновенная скорость» на повседневном языке часто называют просто скоростью.Подчеркните важность использования правильной терминологии в науке, чтобы избежать путаницы и правильно передавать идеи.

Рис. 2.8 За 30 минут до магазина туда и обратно общее расстояние составляет 6 км. Средняя скорость 12 км / ч. Смещение для обхода равно нулю, потому что не было чистого изменения положения.

Рабочий пример

Расчет средней скорости

Мрамор катится 5,2 м за 1,8 с. Какая была средняя скорость мрамора?

Стратегия

Мы знаем расстояние, которое проходит мрамор, 5.2 м, интервал времени 1,8 с. Мы можем использовать эти значения в уравнении средней скорости.

Решение

vavg = расстояние время = 5,2 м 1,8 с = 2,9 м / с vavg = расстояние время = 5,2 м 1,8 с = 2,9 м / с

Обсуждение

Средняя скорость — это скаляр, поэтому мы не включаем направление в ответ. Мы можем проверить разумность ответа, оценив: 5 метров разделить на 2 секунды — это 2,5 м / с. Поскольку 2,5 м / с близко к 2,9 м / с, ответ разумный. Речь идет о скорости быстрой ходьбы, так что это тоже имеет смысл.

Практические задачи

8.

Питчер перебрасывает бейсбольный мяч от насыпи питчера к своей тарелке за 0,46 с. Дистанция 18,4 м. Какая была средняя скорость бейсбольного мяча?

  1. 40 м / с
  2. -40 м / с
  3. 0,03 м / с
  4. 8,5 м / с
9.

Кэсси шла к дому своей подруги со средней скоростью 1,40 м / с. Расстояние между домами 205 м. Как долго она продолжала путешествие?

  1. 146 с
  2. 0.01 с
  3. 2,50 мин
  4. 287 с
Скорость

Векторная версия скорости — это скорость. Скорость описывает скорость и направление объекта. Как и в случае со скоростью, полезно описывать либо среднюю скорость за период времени, либо скорость в определенный момент. Средняя скорость — это смещение, деленное на время, в течение которого смещение происходит.

vavg = время смещения = ΔdΔt = df − d0tf − t0vavg = время смещения = ΔdΔt = df − d0tf − t0

Скорость, как и скорость, выражается в единицах СИ в метрах в секунду (м / с), но поскольку это вектор, вы также должны включить направление.Кроме того, переменная v для скорости выделена жирным шрифтом, потому что это вектор, в отличие от переменной v для скорости, которая выделена курсивом, потому что это скалярная величина.

Советы для успеха

Важно помнить, что средняя скорость — это не то же самое, что средняя скорость без направления. Как мы видели в предыдущем разделе со смещением и расстоянием, изменение направления во временном интервале больше влияет на скорость и скорость.

Предположим, что пассажир двигался к задней части самолета со средней скоростью –4 м / с. Мы не можем сказать по средней скорости, остановился ли пассажир на мгновение или отступил, прежде чем он добрался до задней части самолета. Чтобы получить более подробную информацию, мы должны рассмотреть меньшие сегменты поездки за меньшие промежутки времени, такие как те, что показаны на рисунке 2.9. Если вы рассматриваете бесконечно малые интервалы, вы можете определить мгновенную скорость, которая является скоростью в определенный момент времени.Мгновенная скорость и средняя скорость одинаковы, если скорость постоянна.

Рис. 2.9. На диаграмме представлена ​​более подробная запись пассажира самолета, направляющегося к задней части самолета, показаны более мелкие отрезки его полета.

Ранее вы читали, что пройденное расстояние может отличаться от величины смещения. Точно так же скорость может отличаться от величины скорости. Например, вы едете в магазин и через полчаса возвращаетесь домой.Если одометр вашего автомобиля показывает, что общее пройденное расстояние составило 6 км, то ваша средняя скорость составила 12 км / ч. Однако ваша средняя скорость была равна нулю, потому что ваше смещение в оба конца равно нулю.

Watch Physics

Расчет средней скорости или скорости

В этом видео рассматриваются векторы и скаляры и описывается, как рассчитать среднюю скорость и среднюю скорость, когда вы знаете смещение и изменение во времени. В видео также рассказывается, как преобразовать км / ч в м / с.

Проверка захвата

Что из следующего полностью описывает вектор и скалярную величину и правильно дает пример каждого из них?

  1. Скалярная величина полностью описывается своей величиной, в то время как вектор нуждается как в величине, так и в направлении, чтобы полностью описать его.Смещение — это пример скалярной величины, а время — пример векторной величины.
  2. Скалярная величина полностью описывается своей величиной, в то время как вектор нуждается как в величине, так и в направлении, чтобы полностью описать его. Время — это пример скалярной величины, а смещение — пример векторной величины.
  3. Скалярная величина полностью описывается своей величиной и направлением, тогда как вектору нужна только величина, чтобы полностью описать его.Смещение — это пример скалярной величины, а время — пример векторной величины.
  4. Скалярная величина полностью описывается своей величиной и направлением, тогда как вектору нужна только величина, чтобы полностью описать его. Время — это пример скалярной величины, а смещение — пример векторной величины.

Teacher Support

Teacher Support

Это видео хорошо подчеркивает разницу между векторами и скалярами.Студент знакомится с идеей использования «s» для обозначения смещения, которое вы можете поощрять, а можете и не поощрять. Перед тем, как ученики посмотрят видео, укажите, что преподаватель использует s → s → для смещения вместо d, как в этом тексте. Объясните, что использование маленьких стрелок над переменными является обычным способом обозначения векторов в курсах физики более высокого уровня. Предупредите учащихся, что в этом видео не используются обычные сокращения для часа и секунды. Напомните учащимся, что в своей работе они должны использовать сокращения h для часа и s для секунд.

Рабочий пример

Расчет средней скорости

Учащийся перемещается на 304 м к северу за 180 с. Какая была средняя скорость ученика?

Стратегия

Мы знаем, что смещение составляет 304 м к северу, а время — 180 с. Мы можем использовать формулу для средней скорости, чтобы решить задачу.

Решение

vavg = ΔdΔt = 304 м180 с = 1,7 м / с на север vavg = ΔdΔt = 304 м180 с = 1,7 м / с на север

2,1

Обсуждение

Поскольку средняя скорость является векторной величиной, вы должны включить в ответ направление и величину.Обратите внимание, однако, что направление можно не указывать до конца, чтобы не загромождать проблему. Обратите внимание на значащие цифры в задаче. Расстояние 304 м имеет три значащих цифры, а временной интервал 180 с — только две, поэтому частное должно иметь только две значащие цифры.

Советы для успеха

Обратите внимание на способ представления скаляров и векторов. В этой книге d обозначает расстояние и перемещение. Точно так же v обозначает скорость, а v обозначает скорость.Переменная, которая не выделена жирным шрифтом, указывает на скалярную величину, а выделенная жирным шрифтом переменная указывает на векторную величину. Иногда векторы представлены маленькими стрелками над переменной.

Teacher Support

Teacher Support

Используйте эту задачу, чтобы подчеркнуть важность использования правильного количества значащих цифр в вычислениях. Некоторые студенты имеют тенденцию включать много цифр в свои окончательные вычисления. Они ошибочно полагают, что повышают точность своего ответа, записывая многие цифры, указанные на калькуляторе.Обратите внимание на то, что это приведет к ошибкам в расчетах. В более сложных расчетах эти ошибки могут распространяться и приводить к неправильному окончательному ответу. Вместо этого напомните учащимся всегда носить с собой одну или две дополнительные цифры в промежуточных вычислениях и округлять окончательный ответ до правильного количества значащих цифр.

Рабочий пример

Решение для смещения, когда известны средняя скорость и время

Лейла бегает трусцой со средней скоростью 2.4 м / с на восток. Каково ее смещение через 46 секунд?

Стратегия

Мы знаем, что средняя скорость Лейлы составляет 2,4 м / с на восток, а временной интервал составляет 46 секунд. Мы можем изменить формулу средней скорости, чтобы найти смещение.

Решение

vavg = ΔdΔtΔd = vavgΔt = (2,4 м / с) (46 с) = 1,1 × 102 м на восток vavg = ΔdΔtΔd = vavgΔt = (2,4 м / с) (46 с) = 1,1 × 102 м на восток

2,2

Обсуждение

Ответ: примерно в 110 м к востоку, что является разумным смещением для чуть менее минуты бега трусцой.Калькулятор показывает ответ как 110,4 м. Мы решили написать ответ, используя научную нотацию, потому что мы хотели прояснить, что мы использовали только две значащие цифры.

Советы для успеха

Размерный анализ — хороший способ определить, правильно ли вы решили проблему. Запишите расчет, используя только единицы измерения, чтобы убедиться, что они совпадают на противоположных сторонах отметки равенства. В рассмотренном примере у вас
м = (м / с) (с). Поскольку секунды находятся в знаменателе средней скорости и в числителе времени, единица компенсирует, оставляя только m и, конечно же, m = m.

Рабочий пример

Решение для времени, когда известны смещение и средняя скорость

Филипп идет по прямой дорожке от своего дома до школы. Сколько времени ему потребуется, чтобы добраться до школы, если он пройдет 428 м на запад со средней скоростью 1,7 м / с на запад?

Стратегия

Мы знаем, что смещение Филиппа составляет 428 м на запад, а его средняя скорость составляет 1,7 м / с на запад. Мы можем рассчитать время, необходимое для поездки, переписав уравнение средней скорости.

Решение

vavg = ΔdΔtΔt = Δdvavg = 428 м 1,7 м / с = 2,5 × 102 svavg = ΔdΔtΔt = Δdvavg = 428 м 1,7 м / с = 2,5 × 102 с

2,3

Обсуждение

Здесь нам снова пришлось использовать научную запись, потому что ответ мог состоять только из двух значащих цифр. Поскольку время является скаляром, ответ включает только величину, а не направление.

Практические задачи

10.

Дальнобойщик проезжает по прямой трассе 0,25 ч со смещением 16 км к югу.Какова средняя скорость дальнобойщика?

  1. 4 км / ч север
  2. 4 км / ч юг
  3. 64 км / ч север
  4. 64 км / ч юг
11.

Птица летит со средней скоростью 7,5 м / с на восток от ветки к ветке за 2,4 с. Затем он делает паузу перед полетом со средней скоростью 6,8 м / с на восток в течение 3,5 с к другому ответвлению. Каково полное смещение птицы от начальной точки?

  1. 42 м запад
  2. 6 м запад
  3. 6 м на восток
  4. 42 м на восток

Virtual Physics

The Walking Man

В этом симуляторе вы наведете курсор на человека и переместите его сначала в одном направлении, а затем в противоположном.Не отключайте вкладку Введение . Вы можете использовать вкладку Графики после того, как узнаете о графике движения далее в этой главе. Внимательно следите за знаком чисел в полях положения и скорости. Пока не обращайте внимания на поле ускорения. Посмотрите, сможете ли вы сделать положение человека положительным, а скорость — отрицательным. Затем посмотрите, сможете ли вы сделать обратное.

Проверка захвата

Какая ситуация правильно описывает, когда положение движущегося человека было отрицательным, но его скорость была положительной?

  1. Человек движется к 0 слева от 0
  2. Человек движется к 0 справа от 0
  3. Человек движется от 0 слева от 0
  4. Мужчина движется от 0 справа от 0

Teacher Support

Teacher Support

Это мощная интерактивная анимация, которую можно использовать для многих уроков.На этом этапе его можно использовать, чтобы показать, что смещение может быть как положительным, так и отрицательным. Он также может показать, что при отрицательном смещении скорость может быть как положительной, так и отрицательной. Позже с его помощью можно будет показать, что скорость и ускорение могут иметь разные знаки. Настоятельно рекомендуется оставить учащихся на вкладке Введение . Вкладку Charts можно использовать после того, как студенты узнают о графическом движении позже в этой главе.

Проверьте свое понимание

12.

Два бегуна движутся по одному и тому же прямому пути. Они начинаются в одно и то же время и заканчиваются в одно и то же время, но на полпути у них разные мгновенные скорости. Могут ли они иметь одинаковую среднюю скорость для поездки?

  1. Да, потому что средняя скорость зависит от чистого или полного смещения.
  2. Да, потому что средняя скорость зависит от общего пройденного расстояния.
  3. Нет, потому что скорости обоих бегунов должны оставаться одинаковыми на протяжении всего пути.
  4. Нет, потому что мгновенные скорости бегунов должны оставаться такими же на полпути, но могут быть разными в другом месте.
13.

Если вы разделите общее расстояние, пройденное за поездку на автомобиле (определенное одометром), на время поездки, вычисляете ли вы среднюю скорость или величину средней скорости, и при каких обстоятельствах эти две величины одинаковы? ?

  1. Средняя скорость. Оба они одинаковы, когда автомобиль движется с постоянной скоростью и меняет направление.
  2. Средняя скорость. Оба они одинаковы, когда скорость постоянна и автомобиль не меняет своего направления.
  3. Величина средней скорости. Оба варианта одинаковы, когда автомобиль движется с постоянной скоростью.
  4. Величина средней скорости. И то, и другое одинаково, когда машина не меняет своего направления.
14.

Может ли средняя скорость быть отрицательной?

  1. Да, в случаях, когда чистое смещение отрицательное.
  2. Да, если тело постоянно меняет направление во время движения.
  3. Нет, средняя скорость описывает только величину, а не направление движения.
  4. Нет, средняя скорость описывает только величину в положительном направлении движения.

Teacher Support

Teacher Support

Используйте вопросы Check Your Understanding , чтобы оценить, насколько учащиеся достигли целей обучения по разделам. Если учащиеся не справляются с какой-либо конкретной целью, Check Your Understanding поможет определить, кто из них и направит их к соответствующему содержанию.Тестовые задания в TUTOR позволят вам переоценить.

7.2: Работа — Физика LibreTexts

Цели обучения

  • Изобразите работу, выполненную любым усилием
  • Оцените проделанную работу для различных сил

В физике работа представляет собой тип энергии. Работа выполняется, когда сила действует на что-то, что перемещается из одного положения в другое. Силы могут варьироваться в зависимости от положения, а смещения могут происходить по разным путям между двумя точками.Сначала мы определяем приращение работы dW, совершаемой силой \ (\ vec {F} \), действующей через бесконечно малое смещение d \ (\ vec {r} \), как скалярное произведение этих двух векторов:

\ [dW = \ vec {F} \ cdotp d \ vec {r} = | \ vec {F} || d \ vec {r} | \ cos \ theta \ ldotp \ label {7.1} \]

Затем мы можем сложить вклады для бесконечно малых смещений вдоль пути между двумя положениями, чтобы получить общую работу.

Работа, выполняемая силой

Работа, совершаемая силой, представляет собой интеграл силы по отношению к перемещению по пути перемещения:

\ [W_ {AB} = \ int_ {путь \; AB} \ vec {F} \ cdotp d \ vec {r} \ ldotp \ label {7.2} \]

Векторы, участвующие в определении работы, совершаемой силой, действующей на частицу, показаны на рисунке \ (\ PageIndex {1} \).

Рисунок \ (\ PageIndex {1} \): Векторы, используемые для определения работы. Сила, действующая на частицу, и ее бесконечно малое смещение показаны в одной точке на пути между A и B. Бесконечно малая работа является скалярным произведением этих двух векторов; общая работа — это интеграл скалярного произведения вдоль пути.

Мы предпочитаем выражать скалярное произведение в терминах величин векторов и косинуса угла между ними, потому что значение скалярного произведения для работы можно выразить словами более прямо в терминах величин и углов.С таким же успехом мы могли бы выразить точечный продукт в терминах различных компонентов, представленных в Vectors. В двух измерениях это были компоненты x и y в декартовых координатах или компоненты r и \ (\ varphi \) в полярных координатах; в трех измерениях это были только x-, y- и z-компоненты. Какой вариант удобнее, зависит от ситуации. Другими словами, вы можете выразить уравнение \ ref {7.1} для работы, совершаемой силой, действующей над смещением, как произведение одного компонента, действующего параллельно другому компоненту.Исходя из свойств векторов, не имеет значения, берете ли вы составляющую силы, параллельную смещению, или составляющую смещения, параллельную силе — в любом случае вы получите тот же результат.

Напомним, что величина силы, умноженная на косинус угла, который сила составляет с данным направлением, является составляющей силы в данном направлении. Компоненты вектора могут быть положительными, отрицательными или нулевыми, в зависимости от того, составляет ли угол между вектором и направлением компонента от 0 ° до 90 ° или от 90 ° до 180 °, или он равен 90 °.В результате работа, совершаемая силой, может быть положительной, отрицательной или нулевой, в зависимости от того, направлена ​​ли сила обычно в направлении смещения, обычно противоположно смещению или перпендикулярно смещению. Максимальная работа совершается данной силой, когда она находится в направлении смещения (cos \ (\ theta \) = ± 1), а нулевая работа выполняется, когда сила перпендикулярна смещению (cos \ (\ theta \) = 0).

Единицы работы — это единицы силы, умноженные на единицы длины, которые в системе СИ составляют ньютоны на метры, Н • м.Эта комбинация называется джоуль по историческим причинам, о которых мы поговорим позже, и обозначается сокращенно как J. В английской системе, все еще используемой в Соединенных Штатах, единицей силы является фунт (фунт), а единицей измерения расстояния является фут (фут), поэтому единицей работы является фут-фунт (фут • фунт).

Работа, выполняемая постоянными силами и контактными силами

Самая простая работа для оценки — это работа, выполняемая силой, постоянной по величине и направлению. В этом случае мы можем вычесть силу; оставшийся интеграл — это просто полное смещение, которое зависит только от конечных точек A и B, но не от пути между ними:

\ [W_ {AB} = \ vec {F} \ cdotp \ int_ {A} ^ {B} d \ vec {r} = \ vec {F} \ cdotp (\ vec {r} _ {B} — \ vec {r} _ {A}) = | \ vec {F} || \ vec {r} _ {B} — \ vec {r} _ {A} | \ соз \ тета \; (постоянная \; сила) \ ldotp \ nonumber \]

Мы также можем увидеть это, выписав уравнение \ ref {7.{B} dz \\ & = F_ {x} (x_ {B} — x_ {A}) + F_ {y} (y_ {B} — y_ {A}) + F_ {z} (z_ {B} — z_ {A}) = \ vec {F} \ cdotp (\ vec {r} _ {B} — \ vec {r} _ {A}) \ ldotp \ end {split} \ nonumber \]

На рисунке \ (\ PageIndex {2a} \) показано, как человек прилагает постоянную силу \ (\ vec {F} \) к ручке газонокосилки, которая образует угол \ (\ theta \) с горизонталью. Горизонтальное смещение газонокосилки, на которое действует сила, равно \ (\ vec {d} \). На газонокосилке проделано

работ.

\ [W = \ vec {F} \ cdotp \ vec {d} = Fd \ cos \ theta, \ nonumber \]

, который на рисунке также показан как горизонтальная составляющая силы, умноженная на величину смещения.

Рисунок \ (\ PageIndex {2} \): Работа, выполняемая постоянной силой. (а) Человек толкает газонокосилку с постоянной силой. Составляющая силы, параллельная перемещению, — это проделанная работа, как показано в уравнении на рисунке. (б) Человек держит портфель. Работа не выполняется, потому что смещение равно нулю. (c) Человек в (b) ходит горизонтально, держа портфель. Работа не выполняется, потому что cos \ (\ theta \) равен нулю.

На рисунке \ (\ PageIndex {2b} \) показан человек, держащий портфель.Человек должен приложить восходящую силу, равную по величине весу портфеля, но эта сила не работает, потому что смещение, на которое она действует, равно нулю. Так почему же вы в конце концов чувствуете усталость, просто держа портфель, если вы не делаете с ним никакой работы? Ответ заключается в том, что мышечные волокна в вашей руке сокращаются и выполняют работу внутри вашей руки, даже несмотря на то, что сила, которую ваши мышцы прилагают к портфелю извне, не влияет на него. (Часть силы, которую вы прикладываете, также может быть напряжением в костях и связках вашей руки, но другие мышцы вашего тела будут выполнять работу, чтобы сохранить положение вашей руки.)

На рисунке \ (\ PageIndex {2c} \), где человек в (b) идет горизонтально с постоянной скоростью, работа, выполняемая человеком на портфеле, все еще равна нулю, но теперь, потому что угол между прилагаемой силой и смещение составляет 90 ° (\ (\ vec {F} \) перпендикулярно \ (\ vec {d} \)) и cos 90 ° = 0.

Пример \ (\ PageIndex {1} \): расчет работы, которую вы выполняете, чтобы толкнуть газонокосилку

Сколько работы делает на газонокосилке человек, изображенный на рисунке \ (\ PageIndex {2a} \), если он прилагает постоянное усилие в 75 единиц.0 N под углом 35 ° ниже горизонтали и толкает косилку на 25,0 м по ровной поверхности?

Стратегия

Мы можем решить эту проблему, подставив указанные значения в определение работы, выполняемой над объектом с помощью постоянной силы, указанной в уравнении W = Fd cos \ (\ theta \). Даны сила, угол и перемещение, так что неизвестна только работа W.

Решение

Уравнение для работы

\ [W = Fd \ cos \ theta \ ldotp \ nonumber \]

Подстановка известных значений дает

\ [W = (75.{3} \; J \ ldotp \ nonumber \]

Значение

Несмотря на то, что полтора килоджоулей могут показаться большим трудом, мы увидим в разделе «Потенциальная энергия и сохранение энергии», что это примерно столько же работы, сколько вы могли бы сделать, сжигая одну шестую грамма жира.

Когда вы косите траву, на газонокосилку действуют другие силы, помимо силы, которую вы прикладываете, а именно сила контакта земли и сила гравитации Земли. Давайте в целом рассмотрим работу, проделанную этими силами.Для объекта, движущегося по поверхности, смещение d \ (\ vec {r} \) касается поверхности. Часть силы контакта с объектом, которая перпендикулярна поверхности, является нормальной силой \ (\ vec {N} \). Поскольку косинус угла между нормалью и касательной к поверхности равен нулю, имеем

\ [dW_ {N} = \ vec {N} \ cdotp d \ vec {r} = \ vec {0} \ ldotp \ nonumber \]

Нормальная сила никогда не работает в этих условиях. (Обратите внимание, что если бы смещение d \ (\ vec {r} \) действительно имело относительный компонент, перпендикулярный поверхности, объект либо покинул бы поверхность, либо прорвался через нее, и больше не было бы нормальной контактной силы.Однако, если объект больше, чем частица, и имеет внутреннюю структуру, нормальная контактная сила может воздействовать на него, например, перемещая его или деформируя его форму. Об этом будет сказано в следующей главе.)

Часть силы контакта с объектом, параллельная поверхности, — это трение \ (\ vec {f} \). Для этого объекта, скользящего по поверхности, кинетическое трение \ (\ vec {f} _ {k} \) противоположно d \ (\ vec {r} \) относительно поверхности, поэтому работа, совершаемая кинетическим трением, равна отрицательный.{B} | dr | = — f_ {k} | l_ {AB} | \ ldotp \ label {7.3} \]

где | l AB | — длина пути на поверхности. (Обратите внимание, что, особенно если работа, выполняемая силой отрицательная, люди могут ссылаться на работу, проделанную против этой силы, где dW против = −dW на . Работа, проделанная против силы, также может рассматриваться как работа, необходимая для преодоления этой силы, например, «Сколько работы требуется, чтобы преодолеть…?») Сила статического трения, однако, может выполнять положительную или отрицательную работу.Когда вы идете, сила статического трения, оказываемого землей на заднюю ногу, ускоряет вас на часть каждого шага. Если вы замедляетесь, сила земли на передней ноге замедлит вас. Если вы ведете машину на предельной скорости по прямому ровному участку шоссе, отрицательная работа, выполняемая кинетическим трением сопротивления воздуха, уравновешивается положительной работой, выполняемой статическим трением дороги о ведущие колеса. Вы можете вытащить коврик из-под предмета так, чтобы он соскользнул назад по отношению к ковру, но вперед по отношению к полу.В этом случае кинетическое трение, оказываемое ковриком на объект, может быть в том же направлении, что и смещение объекта относительно пола, и выполнять положительную работу. Суть в том, что вам нужно анализировать каждый конкретный случай, чтобы определить работу, совершаемую силами, будь то положительные, отрицательные или нулевые.

Пример \ (\ PageIndex {2} \): перемещение дивана

Вы решили переместить диван на новое место на горизонтальном полу гостиной. Нормальная сила на кушетке составляет 1 кН, а коэффициент трения равен 0.6. (a) Сначала вы толкаете кушетку на 3 м параллельно стене, а затем на 1 м перпендикулярно стене (от A до B на рисунке \ (\ PageIndex {3} \)). Сколько работы совершает сила трения? (б) Вам не нравится новое положение, поэтому вы перемещаете диван прямо в исходное положение (от B до A на рисунке \ (\ PageIndex {3} \)). Какая общая работа была проделана против трения, смещающего диван из исходного положения и обратно?

Рисунок \ (\ PageIndex {3} \): вид сверху путей для перемещения кушетки.

Стратегия

Величина силы кинетического трения на кушетке постоянна, равна коэффициенту трения, умноженному на нормальную силу, f K = \ (\ mu_ {K} \) N.Следовательно, проделанная им работа будет равна W fr = −f K d, где d — длина пройденного пути. Сегменты путей — это стороны прямоугольного треугольника, поэтому длины путей легко вычислить. В части (b) вы можете использовать тот факт, что работа, проделанная против силы, является отрицательной по сравнению с работой, проделанной этой силой.

Решение

  1. Работа, совершаемая трением, равна $$ W = — (0,6) (1 \; кН) (3 \; m + 1 \; m) = — 2,4 \; кДж \ ldotp \ nonumber $$
  2. Длина пути вдоль гипотенузы составляет \ (\ sqrt {10} \) м, поэтому общая работа, проделанная против трения, составляет $$ W = (0.6) (1 \; кН) (3 \; m + 1 \; m + \ sqrt {10} \; m) = 4.3 \; кДж \ ldotp \ nonumber $$

Значение

Общий путь, на котором оценивалась работа трения, начинался и заканчивался в той же точке (это был замкнутый путь), так что полное смещение кушетки было равно нулю. Однако общий объем работы не был нулевым. Причина в том, что силы, подобные трению, классифицируются как неконсервативные силы или диссипативные силы, как мы обсудим в следующей главе.

Упражнение 7.1

Может ли кинетическое трение быть постоянной силой для всех путей?

Другой силой, действующей на газонокосилку, упомянутую выше, была сила тяжести Земли или вес газонокосилки. Вблизи поверхности Земли гравитационная сила, действующая на объект массы m, имеет постоянную величину mg и постоянное направление вертикально вниз. Следовательно, действие силы тяжести на объект является скалярным произведением его веса и смещения. Во многих случаях удобно выразить скалярное произведение гравитационной работы через x-, y- и z-компоненты векторов.В типичной системе координат ось x расположена горизонтально, а ось y — вертикально вверх. Тогда сила тяжести равна −mg \ (\ hat {j} \), поэтому работа, совершаемая гравитацией на любом пути от A до B, равна

.

\ [W_ {грав, \; AB} = -mg \ hat {j} \ cdotp (\ vec {r} _ {B} — \ vec {r} _ {A}) = -mg (y_ {B} — y_ {A}) \ ldotp \ этикетка {7.4} \]

Работа, выполняемая постоянной силой тяжести над объектом, зависит только от веса объекта и разницы в высоте, на которую объект перемещается. Гравитация выполняет отрицательную работу с объектом, который движется вверх (y B > y A ), или, другими словами, вы должны выполнять положительную работу против силы тяжести, чтобы поднять объект вверх.С другой стороны, гравитация оказывает положительное воздействие на объект, который движется вниз (y B A ), или вы выполняете отрицательную работу против силы тяжести, чтобы «поднять» объект вниз, контролируя его спуск, чтобы он не упал на землю. земля. («Подъем» используется вместо «падение».)

Пример \ (\ PageIndex {3} \): полка книги

Вы поднимаете большую библиотечную книгу весом 20 Н на 1 м вертикально вниз с полки и переносите ее на 3 м горизонтально к столу (рисунок \ (\ PageIndex {4} \)).Сколько работы гравитации выполняет над книгой? (б) Когда вы закончите, вы перемещаете книгу по прямой на ее исходное место на полке. Какова была общая работа, проделанная против силы тяжести, когда книга сместилась с исходного положения на полке и вернулась обратно?

Рисунок \ (\ PageIndex {4} \): вид сбоку путей перемещения книги на полку и с полки.

Стратегия

Мы только что видели, что работа, совершаемая постоянной силой тяжести, зависит только от веса перемещаемого объекта и разницы в высоте пройденного пути, W AB = −mg (yB — y A ).Мы можем оценить разницу в росте, чтобы ответить (а) и (б).

Решение

  1. Поскольку книга начинается на полке и опускается вниз y B — y A = −1 м, имеем $$ W = — (20 \; N) (- 1 \; m) = 20 Дж. \ ldotp \ nonumber $$
  2. Нет разницы в высоте для любого пути, который начинается и заканчивается в одном и том же месте на полке, поэтому W = 0.

Значение

Gravity выполняет положительную работу (20 Дж), когда книга опускается с полки.Гравитационная сила между двумя объектами — это сила притяжения, которая совершает положительную работу, когда объекты становятся ближе друг к другу. Гравитация выполняет нулевую работу (0 Дж), когда книга перемещается горизонтально от полки к столу, и отрицательную работу (-20 Дж), когда книга перемещается со стола обратно на полку. Полная работа силы тяжести равна нулю [20 Дж + 0 Дж + (-20 Дж) = 0].

В отличие от трения или других диссипативных сил, описанных в примере \ (\ PageIndex {2} \), общая работа, проделанная против силы тяжести по любому замкнутому пути, равна нулю.Положительная работа выполняется против силы тяжести на восходящих участках замкнутого пути, но равное количество отрицательной работы выполняется против силы тяжести на нисходящих частях. Другими словами, работа, выполненная против силы тяжести , подняв объект вверх на , «возвращается», когда объект возвращается вниз. Такие силы, как гравитация (те, которые не работают на любом замкнутом пути), классифицируются как консервативные силы и играют важную роль в физике.

Упражнение 7.2

Может ли гравитация Земли быть постоянной силой для всех путей?

Работа, выполняемая различными силами

В общем, силы могут различаться по величине и направлению в точках пространства, а пути между двумя точками могут быть искривленными.Бесконечно малая работа, совершаемая переменной силой, может быть выражена в терминах составляющих силы и смещения по траектории,

\ [dW = F_ {x} dx + F_ {y} dy + F_ {z} dz \ ldotp \ nonumber \]

Здесь компоненты силы являются функциями положения вдоль пути, а смещения зависят от уравнений пути. (Хотя мы решили иллюстрировать dW в декартовых координатах, другие координаты лучше подходят для некоторых ситуаций.) Уравнение \ ref {7.2} определяет общую работу как линейный интеграл или предел суммы бесконечно малых объемов работы.Физическая концепция работы проста: вы рассчитываете работу для крошечных перемещений и складываете их. Иногда математика может показаться сложной, но следующий пример демонстрирует, насколько чисто они могут работать.

Пример \ (\ PageIndex {4} \): работа, выполненная переменной силой по изогнутой траектории

Объект движется по параболической траектории y = (0,5 м −1 ) x 2 от начала координат A = (0, 0) до точки B = (2 м, 2 м) под действием силы. \ (\ vec {F} \) = (5 Н / м) y \ (\ hat {i} \) + (10 Н / м) x \ (\ hat {j} \) (Рисунок \ (\ PageIndex { 5} \)).{2}) \ left (\ dfrac {8} {3} \ right) = 33,3 \; J \ ldotp \ nonumber \]

Значение

Этот интеграл сделать несложно. Вы можете выполнить те же шаги, что и в этом примере, чтобы вычислить линейные интегралы, представляющие работу для более сложных сил и путей. В этом примере все было дано в терминах x- и y-компонентов, которые проще всего использовать при оценке работы в этом случае. В других ситуациях значения и углы могут быть проще.

Упражнение \ (\ PageIndex {4} \)

Найдите работу, проделанную той же силой в Примере \ (\ PageIndex {4} \) над кубическим путем, y = (0.25 м −2 ) x 3 , между теми же точками A = (0, 0) и B = (2 м, 2 м).

В примере \ (\ PageIndex {4} \) вы видели, что для вычисления линейного интеграла вы можете уменьшить его до интеграла по одной переменной или параметру. Обычно это можно сделать несколькими способами, которые могут быть более или менее удобными в зависимости от конкретного случая. В Примере \ (\ PageIndex {4} \) мы уменьшили линейный интеграл до интеграла по x, но мы также могли бы решить свести все к функции от y.{2/3} = 33,3 \; J \ ldotp \ nonumber \]

Как и ожидалось, это точно такой же результат, как и раньше.

Одна очень важная и широко применяемая переменная сила — это сила, создаваемая идеально упругой пружиной, которая удовлетворяет закону Гука \ (\ vec {F} \) = −k \ (\ Delta \ vec {x} \), где k — жесткость пружины, а \ (\ Delta \ vec {x} \) = \ (\ vec {x} \) — \ (\ vec {x} _ {eq} \) — смещение от нерастянутого (равновесного) положения пружины. {2} \ big) \ ldotp \ label {7.5} \]

Рисунок \ (\ PageIndex {6} \): (a) Пружина не оказывает никакого воздействия в своем положении равновесия. Пружина оказывает силу в направлении, противоположном (b) растяжению или растяжению и (c) сжатию.

Обратите внимание, что W AB зависит только от начальной и конечной точек, A и B, и не зависит от фактического пути между ними, если он начинается в A и заканчивается в B. То есть фактический путь может включать переходя туда и обратно перед окончанием.

Еще одна интересная вещь, на которую стоит обратить внимание по уравнению \ ref {7.5} состоит в том, что для этого одномерного случая вы можете легко увидеть соответствие между работой, совершаемой силой, и площадью под кривой силы в зависимости от ее смещения. Напомним, что в общем случае одномерный интеграл — это предел суммы бесконечно малых f (x) dx, представляющий площадь полос, как показано на рисунке \ (\ PageIndex {7} \). В уравнении \ ref {7.5}, поскольку F = −kx — прямая линия с наклоном −k, при построении графика зависимости от x «площадь» под линией представляет собой просто алгебраическую комбинацию треугольных «областей», где «области» выше оси x положительны, а оси ниже — отрицательны, как показано на рисунке \ (\ PageIndex {8} \).Величина одной из этих «площадей» равна половине основания треугольника по оси x, умноженной на высоту треугольника по оси силы. (Термин «площадь» заключен в кавычки, потому что это изделие базовой высоты имеет единицы работы, а не квадратные метры.)

Рисунок \ (\ PageIndex {7} \): кривая зависимости f (x) от x, показывающая площадь бесконечно малой полосы, f (x) dx, и сумму таких площадей, которая является интегралом от f (x) от x 1 до x 2 . Рисунок \ (\ PageIndex {8} \): Кривая силы пружины f (x) = −kx в зависимости от x, показывая области под линией, между x A и x B , для положительных и отрицательных значений x A .Когда x A отрицательное, общая площадь под кривой интеграла в уравнении \ ref {7.5} является суммой положительных и отрицательных треугольных площадей. Когда x A положительный, общая площадь под кривой равна разнице между двумя отрицательными треугольниками.

Пример \ (\ PageIndex {5} \): работа, выполненная усилием пружины

Совершенно упругой пружине требуется 0,54 Дж работы, чтобы растянуться на 6 см от положения равновесия, как показано на рисунке \ (\ PageIndex {6b} \). (а) Какова ее жесткость пружины k? б) Сколько работы потребуется, чтобы растянуть его еще на 6 см?

Стратегия

«Требуемая» работа означает работу, совершаемую против силы пружины, которая является отрицательной по отношению к работе в уравнении \ ref {7.{2}], \; так\; к = 1,62 \; J \ ldotp \ nonumber $$

Значение

Поскольку работа, выполняемая силой пружины, не зависит от пути, вам нужно было только рассчитать разницу в количестве \ (\ frac {1} {2} \) kx 2 в конечных точках. Обратите внимание, что работа, необходимая для растяжения пружины от 0 до 12 см, в четыре раза больше, чем требуется для ее растяжения от 0 до 6 см, потому что эта работа зависит от квадрата величины растяжения от равновесия, \ (\ frac {1} {2} \) кх 2 .В этом случае работа по растяжению пружины от 0 до 12 см также равна работе для составной траектории от 0 до 6 см с последующим дополнительным растяжением от 6 до 12 см. Следовательно, 4W (от 0 до 6 см) = W (от 0 до 6 см) + W (от 6 до 12 см) или W (от 6 до 12 см) = 3W (от 0 до 6 см), как мы найдено выше.

Упражнение \ (\ PageIndex {5} \)

Пружина в примере \ (\ PageIndex {5} \) сжимается на 6 см от своей равновесной длины. (а) Выполняет ли сила пружины положительную или отрицательную работу и (б) какова величина?

Авторы и авторство

  • Сэмюэл Дж.Линг (Государственный университет Трумэна), Джефф Санни (Университет Лойола Мэримаунт) и Билл Мобс со многими авторами. Эта работа лицензирована OpenStax University Physics в соответствии с лицензией Creative Commons Attribution License (4.0).

Расстояние, скорость и ускорение

Основные отношения

Напомним из нашего исследования производных, что если \ (x \ left (t \ right) \) — это положение объекта, движущегося по прямой в момент времени \ (t, \), то скорость объекта равна

\ [v \ left (t \ right) = \ frac {{dx}} {{dt}}, \]

, а ускорение —

. {{t_2}} {\ left | {v \ left (t \ right)} \ right | dt}.2}}} {2}. \]


Решенные проблемы

Щелкните или коснитесь проблемы, чтобы увидеть решение.

Пример 1

Скорость объекта задается уравнением \ (v \ left (t \ right) = \ sqrt {4 + t}, \), где скорость \ (v \) измеряется в \ (\ large {\ frac { \ text {m}} {\ text {s}}} \ normalsize, \) время \ (t \) измеряется в секундах. Найдите расстояние, пройденное объектом за первые \ (5 \, \ text {sec}. \)

Пример 2

Частица движется по прямой в положительном направлении оси \ (x — \) со скоростью, заданной уравнением \ (v = 2 \ sqrt {x} \, \ left ({\ large {\ frac {\ text {m}} {\ text {s}}} \ normalsize} \ right).\) Предполагая, что \ (x \ left ({t = 0} \ right) = 0, \) найдите время \ (t \), которое требуется частице, чтобы покрыть первый \ (100 \, \ text {m} \) пути.

Пример 3

Начиная с момента времени \ (t = 0, \) объект движется по прямой со скоростью \ (v \ left (t \ right) = 6 — 2t. 2}}}} \ normalsize} \ right).\) Найдите расстояние, пройденное частицей за \ (3 \ text {rd} \) секунду.

Пример 5

Объект движется по прямой с ускорением, равным \ [a \ left (t \ right) = 1 + \ cos \ left ({\ pi t} \ right). \] Предполагая, что начальная скорость в точке \ (t = 0 \) равна нулю, найдите общее расстояние, пройденное за первую секунду.

Пример 6

Частица начинает движение из состояния покоя с постоянным ускорением. За \ (1 \ text {st} \) секунду он покрывает \ ({d_1} \) метров.2}}}} \ normalsize} \ right) \) на \ ({T_1} = 5 \) секунд. Затем он равномерно замедляется на \ ({T_2} = 10 \) секунд и останавливается. Какое расстояние проходит частица?

Пример 10

Мяч, брошенный вертикально, проходит вдвое большее расстояние за \ (1 \ text {st} \) секунду, чем за \ (2 \ text {nd} \) секунду. Какая максимальная высота достигается мячом?

Пример 1.

Скорость объекта задается уравнением \ (v \ left (t \ right) = \ sqrt {4 + t}, \), где скорость \ (v \) измеряется в \ (\ large {\ frac { \ text {m}} {\ text {s}}} \ normalsize, \) время \ (t \) измеряется в секундах.3}} — 8} \ right]} = {\ frac {2} {3} \ cdot 19} = {\ frac {{38}} {3} \ приблизительно 12,7 \, \ text {m}} \]

Пример 2.

Частица движется по прямой в положительном направлении оси \ (x — \) со скоростью, заданной уравнением \ (v = 2 \ sqrt {x} \, \ left ({\ large {\ frac {\ text {m}} {\ text {s}}} \ normalsize} \ right). \) Предполагая, что \ (x \ left ({t = 0} \ right) = 0, \) найдите время \ (t \) частица покрывает первые \ (100 \, \ text {m} \) пути.

Решение.

Уравнение движения частицы имеет вид

\ [v = \ frac {{dx}} {{dt}} = 2 \ sqrt x.\]

У нас есть простое дифференциальное уравнение, описывающее положение частицы как функцию времени. Разделив переменные и интегрировав обе стороны, получим:

\ [{\ frac {{dx}} {{\ sqrt x}} = 2dt,} \; \; \ Rightarrow {\ int {\ frac {{dx}} {{\ sqrt x}}} = 2 \ int {dt},} \; \; \ Rightarrow {2 \ sqrt x = 2t + C,} \; \; \ Rightarrow {\ sqrt x = t + C.} \]

Из начального условия \ (x \ left ({t = 0} \ right) = 0 \) следует, что \ (C = 0. 2}}}} \ normalsize} \ right ) \).2}}}} \ normalsize} \ right) \) на \ ({T_1} = 5 \) секунд. Затем он равномерно замедляется на \ ({T_2} = 10 \) секунд и останавливается. Какое расстояние проходит частица?

Решение.

Определить скорость \ ({v_1} \) в момент \ (t = {T_1}: \)

\ [{{v_1} = {a_1} t = {a_1} {T_1}} = {5 \ cdot 5} = {25 \, \ left ({\ frac {\ text {m}} {\ text {s }}} \ right)} \]

На втором этапе скорость частицы изменяется согласно уравнению

\ [v = {v_1} + {a_2} t, \]

где \ (0 \ le t \ le {T_2}.2}}} {2}} = {125 \, \ text {m}} \]

Следовательно, полное расстояние, пройденное частицей, равно

\ [{d = {d_1} + {d_2}} = {62,5 + 125} = {187,5 \, \ text {m}} \]

Пример 10.

Мяч, брошенный вертикально, проходит вдвое большее расстояние за \ (1 \ text {st} \) секунду, чем за \ (2 \ text {nd} \) секунду. Какая максимальная высота достигается мячом?

Решение.

Предположим, что мяч подброшен вверх с начальной скоростью \ ({v_0} \).